PDA

View Full Version : I cant decide on which Medical field of study



ICANHITHIMMAN
09-05-11, 12:45
Hay guys I'm at a cross roads in my schooling. I'm 3 years out of the mil and have been going to school for a few years now. I want to go into the medical field but I cant decide which diciploine to focus and I need to start applying to programs.

I'm considering Nurse Practitioner or Physician Assistant I know some of you here are EMTs, Medics, Corpsman, PAs and Doctors can you tell me what you think? I want to do more trama related stuff so whats the best choice.

I dont want to go to medical school right now, I'm not even considering it at this point. I live in Rochester NY so there is no shortage of great schools in the area and with the GI bill its a no brainer.

Jon

Kegger
09-05-11, 15:11
As a medic I have always aspired to be a PA.

For NP, you need to go to nursing school, then on to NP. Too much redundancy.

For PA, just get a bachelors and apply to school. You can continue with your current major and not have to take a bunch of classes that just add another year or two onto your schooling.

But that's just me, YMMV.

kiwi57
09-05-11, 15:32
ICANHITHIMMAN

As usual, it depends. What are your professional goals?

I was a DUSTOFF medic (91B2F) from 1977-1980 in West Germany and started PA school in 1981. Thirty years later, I am grateful to be a PA and could not imagine doing anything else. Getting into school is a real challenge (the school where I teach has 780 applicants for 44 seats this year).

I'd suggest devising a pro and con list after carefully researching the various professions open to you with your military experience (presumably medical) and further education. This would include talking with people who have been in the professions for a few years. An important question to ask is: "Would you do it all over again knowing what you know now?

http://www.aapa.org/
http://www.paeaonline.org/

Good Luck

Kiwi57

Hmac
09-05-11, 15:44
Physician Assistant. Licensure in most states allows for a much broader scope of practice than Nurse Practicioner and you'll have much more flexibility and have many more options available to you. As mentioned, you need a Bachelor's degree in something, preferable science-related. I'd recommend finding a couple of good PA programs and go talk with a counselor - get recommendations as to how best to prepare your education for that track and how to make your application stand out. Consider a BS degree following a pre-med course of study. It may help you if you later decide to go to medical school.

The Physician Assistant concept was initiated to provide a civilian role for all the talented and experienced military coming out of the Viet Nam era. There's still a soft spot in their hearts for ex-military.

If and when you do go the PA route, consider a specialty track. There are some post-graduate fellowship programs after PA training for specializing in some areas of medicine, especially in some of the surgical specialities and Emergency Medicine. The demand in Orthopedics is especially high. Our orthopedics department has been looking hard for an Ortho PA...they're hard to come by and salaries start around $100,000.

norinco982lover
09-05-11, 15:51
I say PA. Might as well get started on your BA now!

GIJew766
09-05-11, 16:04
It depends on what you want. When I got out of the Fleet, I'd have never believed you if you told me I would go into any medical field whatsoever. Yet, a few years later, after doing the whole CBP thing, here I am. I worked a 911 ambulance service as an EMT, then did Medic, working in a hospital as an ER Tech while going to school to prep for med school. I want to do trauma and emergency medicine. For me, this choice works.

For you, if you want to do some trauma, you can do it as a tech, but you can also do it as a nurse or PA. It comes down to how much time you want to invest, and what end result you're chasing. For some, taking the time away from working is a difficult sacrifice. Others work while in school. It comes down to how much time, money and all that you're willing to part with.

No matter what, the medical field is probably one of the best to get into right now. A DoL report a while back said something about the field expects to see 16-19% growth over the next few years, so it's a good time to get in.

Hope this helps.


H

ICANHITHIMMAN
09-05-11, 19:09
Thanks guys

RIT and UofR both have PA programs and I have already geared my current degree in the science/bio/med direction. So I'm on track for any one that I pic right now. There are alot more school options for Nursing in the area.

Thaks for the links and the advice I suppose I will ask the NPs and PAs at my doctors office the questions you poised to me.

I didnt do anything medical in the military beyond CLS traning. I just like helping people and when ever someone went down or needed stiches, bandages etc I liked to watch and help when allowed.

kiwi57
09-05-11, 20:11
Sounds like you have a personality that includes altruism which in my estimation is the best starting point for any health career.

I'm usually reluctant to give too much advice, but you might consider getting an EMT certification. In addition to didactic training, EMT usually includes practical clinical observation with local EMS caregivers. Many colleges and universities either include EMT courses in their allied health departments or give course credit for community programs. Might be a way to get your feet wet while making progress toward your BA/BS degree. Although most PA programs no longer require 3-5 years of full time medical/nursing experience before admission, they do value it, so it is helpful to say "I'm an EMT". Might also open the door to part time EMS or ER jobs. You will be exposed to many disciplines in the ER which may help you sort things out.

Consider "shadowing" a NP, PA, doc, or other provider if they will let you. Privacy issues can be rightfully cumbersome at times. Possessing a medical "credential" such as an EMT cert. may make this somewhat easier.

medicman816
09-05-11, 20:43
As a paramedic, I will post a reply. I will also begin by stating that I am in Pennsylvania. Things may be different in your state. The prehospital field can be interesting. Unless you are in a big city or near a major interstate, don't expect to see much trauma. I service an area of about 15,000 people, more than 50% of which are 65 or older. I can honestly say that I get a "true" trauma patient maybe four times a year. Where I am at it's mostly a taxi service, sorry to say. Salaries suck for the most part.
In PA, a PA can function in the prehospital environment, as can an RN, CRNP, or MD. While an EMT or medic cert may get you some experience and help pay bills while you go to school, I must say that it currently is a poor career choice. If I had it to do again I would still become a medic, but I would also have busted ass to get into a PA or RN school. I would be done by now making three times what I currently make. Plus, I could still do the prehospital thing WHEN I WANTED.
The advice about job shadowing is right on. I would bet that you could walk into your local ambulance or fire station and they would let you ride along with them for however long you wanted. Sign some papers and off you go. There are a lot of people that get into the field this way.
Ok, this post is long enough. If you would like to know anything else about prehospital medicine, shoot me a PM.

ICANHITHIMMAN
09-05-11, 20:56
altruism I had to look that up. But yes thats me in a nut shell just feels right. I'm going to look into the shadowing I have a friend who is a anistegeolist he can set something up for me.

bpw
09-05-11, 21:51
In many states Nurse Practitioners actually have a broader scope of practice with more autonomy than PA's. But as stated above it takes longer for schooling if you are not already a nurse. Starting this year or next many programs are going to a PhD as entry level so figure 4 years plus RN school which would be 4 years for a BSN.

In my hospital most of the NP's work in Medicine units whereas many of the PA's work in Surgery services, mainly because of how they were trained (PA's generally follow the medical model).

Another thing to consider is a therapist career such as Physical or Occupational therapist. If you like the "ER rush" respiratory therapy is also something to look into.

I find radiology interesting and am considering that as my new career when the kids are out of college and I get tired of my NP job.

Basic math, chemistry and biology as a start will serve you whatever you end of doing so I would suggest starting there with school and doing some job shadowing.

Good luck.
b

ICANHITHIMMAN
09-06-11, 09:40
In many states Nurse Practitioners actually have a broader scope of practice with more autonomy than PA's. But as stated above it takes longer for schooling if you are not already a nurse. Starting this year or next many programs are going to a PhD as entry level so figure 4 years plus RN school which would be 4 years for a BSN.

In my hospital most of the NP's work in Medicine units whereas many of the PA's work in Surgery services, mainly because of how they were trained (PA's generally follow the medical model).

Another thing to consider is a therapist career such as Physical or Occupational therapist. If you like the "ER rush" respiratory therapy is also something to look into.

I find radiology interesting and am considering that as my new career when the kids are out of college and I get tired of my NP job.

Basic math, chemistry and biology as a start will serve you whatever you end of doing so I would suggest starting there with school and doing some job shadowing.

Good luck.
b

I have heard this about the NP here in NY. I already have all the classes you mentioned out of the way. I have to go get books today and I'm going to pick out the 2+2 program stuff from school to see whats the best choice. I like the idea of less school with the PA route. I heard the PhD is going to take effect for NP here in NY in 5 years.

Thanks alot

Hmac
09-06-11, 09:50
It might be helpful to look up the scope of practice allowed by the states where you might want to live. I don't know about New York specifically. In many states, including this one, NP's scope of practice is determined by statute, PA's scope of practice is generally determined by their supervising physician.

chuckman
09-06-11, 10:30
I was a paramedic for years, including flight, and I was a corpsman, and am in my 8th year of being a RN. If you 'think' you want to do advanced practice, make that your goal and start working for it. RN is great, very portable, allows for active lifestyle, blah, blah, blah...but you 'cap out' pretty quickly.

I work at Duke and we have a metric crap-ton of mid-levels, PAs (what with Duke having had started the PA field and all) and NPs. In fact, the NPs run critical care in some of the ICUs. The pay here for mid-levels is pretty low as there are a million running around (supply and demand), but there a bunch of jobs. In our ED, the PAs do 'real' medicine, the NPs are urgent care-type of practitioners.

I liked being a medic and a flight medic, and a corpsman, for trauma, and get a lot of it in the ED. If I tire of it, I can always go to another unit. That portability of nursing again.

You have a ton of options, none of them bad. Best of luck.

kiwi57
09-07-11, 21:13
Another important thing to consider is that PA education is an average of 27 straight months and pricey. Its structured similar to a medical school model, not a graduate school model. The first 12-15 months is 30+ hours/week of classroom/lab time and the second year is 40-50 hours/week doing rotations and continuity clinics. Surgery/ER/inpatient rotations are even more hours/week.

I mention this as almost all schools strongly discourage working. I've had students with radiology or respiratory therapy backgrounds who managed to squeeze in a few on-call shifts per month to bring in extra money and they could get some sleep or do a bit of studying. I now have a PA student who is a pediatric ICU RN who tried to do the same but it has not been a good idea--obviously no time to study or rest.

What is the duration of VA benefits nowadays? In my day, "benefit extensions" were intermittently available beyond the standard 4 years. I was able to get great Vietnam-era benefits by volunteering delayed-entry in 1976. In 1977 the benefit structure changed significantly--for the worse. I hope the current VA system is more generous now and covers you through grad school.

JDest
09-08-11, 17:34
I am NOT in the medical field but I have a personal friend who is training to be a PA and loves it. My opinion certainly doesn't carry nearly the weight as some of those above me, but it sounds to me like the route you should choose. Good luck.

ICANHITHIMMAN
09-08-11, 21:13
Another important thing to consider is that PA education is an average of 27 straight months and pricey. Its structured similar to a medical school model, not a graduate school model. The first 12-15 months is 30+ hours/week of classroom/lab time and the second year is 40-50 hours/week doing rotations and continuity clinics. Surgery/ER/inpatient rotations are even more hours/week.

I mention this as almost all schools strongly discourage working. I've had students with radiology or respiratory therapy backgrounds who managed to squeeze in a few on-call shifts per month to bring in extra money and they could get some sleep or do a bit of studying. I now have a PA student who is a pediatric ICU RN who tried to do the same but it has not been a good idea--obviously no time to study or rest.

What is the duration of VA benefits nowadays? In my day, "benefit extensions" were intermittently available beyond the standard 4 years. I was able to get great Vietnam-era benefits by volunteering delayed-entry in 1976. In 1977 the benefit structure changed significantly--for the worse. I hope the current VA system is more generous now and covers you through grad school.

The GI bill pays for 48 months but I'm not using it yet. I have been using VTAP through the state of NY for my under grad schooling. I'm trying to stretch it by working summers as a Union Ironworker and other programs through the state before I tap in to the GI fund. My wife is already throuh school and into her carier so that helps alot.

Gutshot John
09-08-11, 21:29
I'd consider PA more than others.

1. Your malpractice insurance is covered under the doctor's you work for

2. Far more demand for your skill set so excellent job security

3. Less time in school, no internship or residency and many PAs get their loans paid back. Many PA programs include a bachelor's/MS joint program

4. Huge demand for PAs in the mil if you decide to go back in. A corpsman buddy of mine who became a PA after he got out, went back into the Army as an officer and is now serving with the 160th SOAR.

5. Depending on what you like...the upside is that you get to spend lots of time with your patients...the downside is that you get to spend lots of time with your patients.

6. As an ex-corpsman and current paramedic...I much prefer dealing with PAs when I get medical attention

ballsout
09-08-11, 23:06
As a paramedic, I will post a reply. I will also begin by stating that I am in Pennsylvania. Things may be different in your state. The prehospital field can be interesting. Unless you are in a big city or near a major interstate, don't expect to see much trauma. I service an area of about 15,000 people, more than 50% of which are 65 or older. I can honestly say that I get a "true" trauma patient maybe four times a year. Where I am at it's mostly a taxi service, sorry to say. Salaries suck for the most part.
In PA, a PA can function in the prehospital environment, as can an RN, CRNP, or MD. While an EMT or medic cert may get you some experience and help pay bills while you go to school, I must say that it currently is a poor career choice. If I had it to do again I would still become a medic, but I would also have busted ass to get into a PA or RN school. I would be done by now making three times what I currently make. Plus, I could still do the prehospital thing WHEN I WANTED.
The advice about job shadowing is right on. I would bet that you could walk into your local ambulance or fire station and they would let you ride along with them for however long you wanted. Sign some papers and off you go. There are a lot of people that get into the field this way.
Ok, this post is long enough. If you would like to know anything else about prehospital medicine, shoot me a PM.

Medicman have you thought about getting your Paramedic to RN bridge? 1 year program if you have your EMTP. There seems to be a plentiful of Firefighter/EMTPs who go on to also get there RN through the bridge programs.

ballsout
09-08-11, 23:13
ICANHITHIMMAN if your interested you could get your EMTB while taking your prerequisites to RN or PA school. That way you could work as an EMT Tech in an ER get your fill and also be on your way to your goal. Plus it will give you experience and familiarity with ER. You'll have way more hands on patient care hours before anybody in your classes even touches a patient. I believe EMT Techs at hospitals get paid more and are exposed to more than the ambulance scene as a basic.

ICANHITHIMMAN
09-09-11, 12:49
ICANHITHIMMAN if your interested you could get your EMTB while taking your prerequisites to RN or PA school. That way you could work as an EMT Tech in an ER get your fill and also be on your way to your goal. Plus it will give you experience and familiarity with ER. You'll have way more hands on patient care hours before anybody in your classes even touches a patient. I believe EMT Techs at hospitals get paid more and are exposed to more than the ambulance scene as a basic.

I will look into it but to be honest I cant do both right now not enough time in the day.

Thanks to everyone for the continued advice. I have been thinking hard about everything you guys have said. I didnt get the best schedule this semester so time is super tight. The classes I'm taking are all high in demad so I had to pick crazy times and I'm here 3 days a week from 7am to 9pm the other 4 are for homework and life.

snackgunner
09-09-11, 13:00
Not sure if it was mentioned here yet, but incase you didnt know, as of October 1st the post 9/11 GI bill will pay for non-degree programs such as EMT,Paramedic, or Fire Fighter schools.

And Im almost certain, that the Post 9/11 GI Bill is only for 36 months, as opposed to 48. If you qualify, you could apply for Vocational Rehabilitation and they would pay for 12 months of your schooling, making a total of 48 months.

I found this out when calling GI Bill 800 number. I called to see how many months I had left, and they told me the Post 9/11 GI Bill was only good for 36 months. I remember hearing 48 months when I was the Marines and paying into the Montgomery GI Bill. So maybe the Montgomery GI Bill is 48 months long, I dont know. But I found the newer Post 9/11 GI Bill to be better overall than the Montgomery GI Bill.

Good luck with whatever you decide to do.

Semper Fi

extrication
09-11-11, 15:55
After thirty years, flight nurses still impress me the most. It's not just the tight flight suits! The nurses with pediatric specialties are always a welcomed site on remote scenes with little vics involved. It's more dangerous than working an inner city trauma center but boredom is rare.

chuckman
09-11-11, 16:49
After thirty years, flight nurses still impress me the most. It's not just the tight flight suits! The nurses with pediatric specialties are always a welcomed site on remote scenes with little vics involved. It's more dangerous than working an inner city trauma center but boredom is rare.

Well, actually....it IS pretty boring. The scene stuff is exciting and what I enjoyed, but scene stuff was 20% of what we did. Ninety percent of the interfacility transports were snooze-fests, though the patients were always sicker than stink. Now, to be a trooper-medic with Maryland State Police, or a similar high-speed service, that would be cool.

Snareman
10-10-11, 23:51
+1 for the PA route. I've been one for over 10 years now and love it. I work in the ER where I get to see all sorts of good stuff.

As has been mentioned, NPs have to pick a track in school and that limits what they can do when they get out. PAs get a more broad education and can change to any speciality at any time (granted you might have a lot to learn when you switch over). NPs are more independent than PAs, although there are some states with pretty liberal PA laws. Really though its just a matter of knowing what you can and can't handle on your own and what you need to call in backup for. Many PA programs have sort of an all in one type deal now where you get your BS and MS all in line.

ICANHITHIMMAN
10-11-11, 09:22
+1 for the PA route. I've been one for over 10 years now and love it. I work in the ER where I get to see all sorts of good stuff.

As has been mentioned, NPs have to pick a track in school and that limits what they can do when they get out. PAs get a more broad education and can change to any speciality at any time (granted you might have a lot to learn when you switch over). NPs are more independent than PAs, although there are some states with pretty liberal PA laws. Really though its just a matter of knowing what you can and can't handle on your own and what you need to call in backup for. Many PA programs have sort of an all in one type deal now where you get your BS and MS all in line.

Thanks for the reply I'm leaning in that direction now. RIT has the program you just mentioned but I need to knock out a few more pre reqs before I apply.

kiwi57
11-09-11, 21:58
ICANHITHIMMAN,

Just an FYI. President Obama is supporting grant funding to help vets transition into PA training upon ETS:

http://www.whitehouse.gov/the-press-office/2011/10/25/fact-sheet-we-cant-wait-obama-administrations-new-initiatives-help-creat

What this means is that within the next 6-12 months the Health Resources and Services Administration (HRSA) will release guidance to assist PA programs who want to write grants to support "bridging" vets into PA school. Much too soon to know how/when these initiatives will take shape or what the scope will be.

Here's the PAEA president's response:

http://www.paeaonline.org/index.php?ht=d/sp/i/130963/pid/130963

Because you've mentioned RIT, you might want to ask their admissions person if their program is aware of this. They might be well-positioned to go after this funding because of their undergrad/grad model of training.

Kiwi57

TehLlama
11-09-11, 23:01
The wife adds a +1 for the PA program. While P911GIB will cover non-degree programs, PA still looks like the winner in the long run.

ICANHITHIMMAN
11-10-11, 07:01
KIWI

Thanks, to say the least. Im going to look in to it today. I know the vet rep at RIT. He has to be the most locked on rep in any of the local schools.

Jon

Jmacken37
01-12-12, 17:26
I think medicine is a neat field; as an ER PA, I'm biased.

If you're young, single, and have the time and aren't concerned about stuck in one specialty, I'd consider medical school. Otherwise, without a nursing degree, PA training is a no-brainer.

Let's face it...physicians are captains of the ship in medicine. They are more comfortable with PA's because we are trained like they were and are tied to a dependent practice model acknowledging physician leadership.

Degree creep is crazy in healthcare. A doctorate to practice as a pharmacist, physical therapist, NP? Seems like blatant "doctor" envy to me.

Jake

ICANHITHIMMAN
01-12-12, 18:59
I think medicine is a neat field; as an ER PA, I'm biased.

If you're young, single, and have the time and aren't concerned about stuck in one specialty, I'd consider medical school. Otherwise, without a nursing degree, PA training is a no-brainer.

Let's face it...physicians are captains of the ship in medicine. They are more comfortable with PA's because we are trained like they were and are tied to a dependent practice model acknowledging physician leadership.

Degree creep is crazy in healthcare. A doctorate to practice as a pharmacist, physical therapist, NP? Seems like blatant "doctor" envy to me.

Jake

Well thanks for responding. After this semester I am applying to Saint John Fisher school of nursing for my BA then depending on how much money I have left in my GI bill I will apply for the masters program. Should have enough have not used it.

WS6
02-16-12, 04:48
Hay guys I'm at a cross roads in my schooling. I'm 3 years out of the mil and have been going to school for a few years now. I want to go into the medical field but I cant decide which diciploine to focus and I need to start applying to programs.

I'm considering Nurse Practitioner or Physician Assistant I know some of you here are EMTs, Medics, Corpsman, PAs and Doctors can you tell me what you think? I want to do more trama related stuff so whats the best choice.

I dont want to go to medical school right now, I'm not even considering it at this point. I live in Rochester NY so there is no shortage of great schools in the area and with the GI bill its a no brainer.

Jon

RN,BSN here. I work Stepdown and SICU. We are a surgical hospital and don't get much trauma, but you don't have to work at a surgical hospital. Work ER at a place like LSU or other state hospital, and you will get your fill of it.

As I am a nurse, and none of the other you have mentioned, obviously I advise nursing if you think you would like it. Talk to a few people in all those professions, etc. and see what jives best with you.

Lots of "clashing of ranks" in a hospital. Just don't get involved in it. Respect everyone, expect (and show that you expect) respect from everyone. This has never failed me yet. Everyone from MD's to house-keeping smiles and says "hey, how are you?" when they see me, unless I see and greet them first.

panzerr
02-16-12, 08:09
Think twice before becoming a PA. Licensure can be a hassle state to state and everything you do has to be signed off by an MD.

Dentistry is a good line of work so long as you don't have chronic thoracolumbar or cervical-thoracic problems (think being hunched over someone day after day). The work evenvironment would be much more laid back than a hospital and you would likely make more money than many MDs.

Chiropractic is what I am going in to and honestly I wouldn't suggest it unless you get the VA to pay for school (that's what I did). Many of my mates are looking at $200,000 in loans and so far the government only forgives loans for MDs, even though DCs are primary care physicians as well (little known fact). You also need to be business savvy as a Chiro because as of yet if a hospital has a DC on staff it is unusual so you will probably end up in private practice....although this is one of the aspects that drew me to the profession. This means if you aren't personable you will likely fail and default on your student loans because people will not like you and thus not seek out your care.

Hmac
02-16-12, 13:13
Think twice before becoming a PA. Licensure can be a hassle state to state and everything you do has to be signed off by an MD.



Maybe it varies state to state. I work with PA's, and while I'm obligated to take responsibility for their work, I certainly don't have to sign off on everything, or even anything, that they do.




Chiropractic .....

:)

kiwi57
02-16-12, 20:59
Re:

". . . After this semester I am applying to Saint John Fisher school of nursing for my BA . . ."


ICANHITHIMMAN,

Congratulations on arriving at a decision—must be a relief. The nice thing about becoming a BSN is that you can choose from a wide variety of settings to practice in after graduation. You can remain an RN, apply to be an advanced practice nurse or try the physician assistant route. Each will be a possibility with a BSN background.

During your training, you will have ample opportunity to talk with many nurses, docs and PAs about your next step. As I mentioned before, one of the best questions to ask people you grow to respect would be: “if you had it to do all over again, would you do the same thing?” If they reply “no”, find out why and what they would have done instead.

Of course, their replies have to be taken with a grain of salt because burn out is not uncommon in some of the health professions. Someone else’s job satisfaction is no guarantee of your own—but keep your ears open for recurring themes. I hope I’m not coming off as preachy, but this has been a helpful question in my experience.

Hmac,

Your comment is spot-on, co-signature and supervision requirements for PAs vary by state. With very few exceptions, state professional licensing rules have improved significantly in the 30 years I’ve been a PA. In Utah, it is up to the doc and the PA to decide. As you know, what’s really important for any clinician is that they understand their limits and feel no hesitation to ask for help when caring for a patient.

Kiwi57

ICANHITHIMMAN
02-16-12, 21:01
Think twice before becoming a PA. Licensure can be a hassle state to state and everything you do has to be signed off by an MD.

Dentistry is a good line of work so long as you don't have chronic thoracolumbar or cervical-thoracic problems (think being hunched over someone day after day). The work evenvironment would be much more laid back than a hospital and you would likely make more money than many MDs.

Chiropractic is what I am going in to and honestly I wouldn't suggest it unless you get the VA to pay for school (that's what I did). Many of my mates are looking at $200,000 in loans and so far the government only forgives loans for MDs, even though DCs are primary care physicians as well (little known fact). You also need to be business savvy as a Chiro because as of yet if a hospital has a DC on staff it is unusual so you will probably end up in private practice....although this is one of the aspects that drew me to the profession. This means if you aren't personable you will likely fail and default on your student loans because people will not like you and thus not seek out your care.

I could never ever ever look in peoples mouths all day. I have a thing now when I speek to people if they dont take care of there teeth I cant even look them in the eye.

ICANHITHIMMAN
02-16-12, 21:02
RN,BSN here. I work Stepdown and SICU. We are a surgical hospital and don't get much trauma, but you don't have to work at a surgical hospital. Work ER at a place like LSU or other state hospital, and you will get your fill of it.

As I am a nurse, and none of the other you have mentioned, obviously I advise nursing if you think you would like it. Talk to a few people in all those professions, etc. and see what jives best with you.

Lots of "clashing of ranks" in a hospital. Just don't get involved in it. Respect everyone, expect (and show that you expect) respect from everyone. This has never failed me yet. Everyone from MD's to house-keeping smiles and says "hey, how are you?" when they see me, unless I see and greet them first.

Thanks for the advice I have heard there is in fighting at some places

Sensei
02-16-12, 22:31
One important and growing aspect of medicine is the notion of patient satisfaction and customer service. Most patients who visit a doctor's office, ER, or have any other contact with a healthcare professional will receive a patient satisfaction survey. These surveys are increasingly used to determine provider compensation, and CMS (Center for Medicare Services) will soon use patient satisfaction scores as a factor in hospital reimbursement for in-patient admissions.

Now, I'm not opposed to measuring or rewarding customer service. However, most of patients are not customers in the usual sense. For example, about 1/4 of my patients have Medicare, 1/4 have Medicaid, 1/3 have private insurance, and the rest have no coverage (or any intention of paying a bill). That means about 70% of the people who walk in the ER have no personal incentive to ration their care because someone else is paying for all or most of their bill. Yet, all of them will get a survey, and their response will influence the provider's compensation. In other words, these surveys largely measure consumer rather than customer satisfaction in that the consumers have no stake in the game. Imagine if Walmart gave a customer service survey to shoplifters (the equivalevt of giving a survey to someone who steals a provider's intellectual property by not paying a bill).

This has a very detrimental effect on resource utilization, over-prescription of antibiotics, and inappropriate use of controlled substances. For example, are you really not going to order that head CT for the kid with a head bump brought by a neurotic mother, or not prescribe antibiotics for a kid's cold after the mom says she thinks he needs a Z-pack? Well, the answer to both of those patients is "no" - but you better have the social skills to do it in a way that does not cost you a lot of money.

This means that medicine is increasingly becoming more of a customer service industry and less of a technical field. None of my partners fret over malpractice; it is the emerging customer service aspects of medicine that gives most of us ulcers. You better understand that before you committ to the personal and financial sacrifice.

ICANHITHIMMAN
02-17-12, 06:40
Wow good info and thanks for sharing

WS6
02-17-12, 07:13
One important and growing aspect of medicine is the notion of patient satisfaction and customer service. Most patients who visit a doctor's office, ER, or have any other contact with a healthcare professional will receive a patient satisfaction survey. These surveys are increasingly used to determine provider compensation, and CMS (Center for Medicare Services) will soon use patient satisfaction scores as a factor in hospital reimbursement for in-patient admissions.

Now, I'm not opposed to measuring or rewarding customer service. However, most of patients are not customers in the usual sense. For example, about 1/4 of my patients have Medicare, 1/4 have Medicaid, 1/3 have private insurance, and the rest have no coverage (or any intention of paying a bill). That means about 70% of the people who walk in the ER have no personal incentive to ration their care because someone else is paying for all or most of their bill. Yet, all of them will get a survey, and their response will influence the provider's compensation. In other words, these surveys largely measure consumer rather than customer satisfaction in that the consumers have no stake in the game. Imagine if Walmart gave a customer service survey to shoplifters (the equivalevt of giving a survey to someone who steals a provider's intellectual property by not paying a bill).

This has a very detrimental effect on resource utilization, over-prescription of antibiotics, and inappropriate use of controlled substances. For example, are you really not going to order that head CT for the kid with a head bump brought by a neurotic mother, or not prescribe antibiotics for a kid's cold after the mom says she thinks he needs a Z-pack? Well, the answer to both of those patients is "no" - but you better have the social skills to do it in a way that does not cost you a lot of money.

This means that medicine is increasingly becoming more of a customer service industry and less of a technical field. None of my partners fret over malpractice; it is the emerging customer service aspects of medicine that gives most of us ulcers. You better understand that before you committ to the personal and financial sacrifice.

I think that the ultimate thing to remember is:

People what what they think is best for themselves.
The goal of healthcare is to teach people what is best for themselves--in a friendly, non-confrontational manner, which is most effective when the patient feels that they themselves have, by their own wit, come to the conclusion that you have steered them toward.

A healthcare provider that can manage this, be it a nurse like me, or a PA like others, or an MD like others, will always have an easier go at things.

A lot of regulation in healthcare is aimed at preventing "lowest common denominator" type mistakes. It's only going to get more confusing, more red-tape oriented, and ultimately more costly. Think about this before becoming a private-practice MD.

montanadave
02-17-12, 07:30
Now, I'm not opposed to measuring or rewarding customer service. However, most of patients are not customers in the usual sense. For example, about 1/4 of my patients have Medicare, 1/4 have Medicaid, 1/3 have private insurance, and the rest have no coverage (or any intention of paying a bill). That means about 70% of the people who walk in the ER have no personal incentive to ration their care because someone else is paying for all or most of their bill. Yet, all of them will get a survey, and their response will influence the provider's compensation. In other words, these surveys largely measure consumer rather than customer satisfaction in that the consumers have no stake in the game. Imagine if Walmart gave a customer service survey to shoplifters (the equivalevt of giving a survey to someone who steals a provider's intellectual property by not paying a bill).



While I have long been aware of this distinction, I have never read it phrased in quite that way. Very clearly cuts right to the heart of one of the critical disconnects which is crippling health care.

To the OP, I found myself in almost exactly your position about 20 years ago when I was confronted with making a mid-life career change (I was not military, however). After completing my EMT training, I had decided to go back to college to get a second degree to allow me to get into health care. My original intention was to complete the necessary pre-requisites to facilitate my acceptance into a PA program (there were far fewer at that time and placement was tight). As I worked my way through the undergraduate healthcare track curriculum (microbiology, A&P, organic chem., etc.), I found myself in more and more classes with nursing students completing their first two years of a four year BSN program. I ultimately decided to get my BSN, get some practical experience, and then make a decision about pursuing a mid-level provider degree. I rationalized that decision by analogizing my RN licensure to a journeyman's license allowing me to find a good-paying job just about anywhere I might find myself. And that same logic holds true today, perhaps even more so.

After spending several years working in the hospital environment, I transitioned into home health care case management. This led me to getting a graduate degree and licensure in the mental health care field. All of this is to say that health care is a BIG area with a wide variety of careers, specialities, work environments, and employment options.

I don't think you will regret getting a BSN. Nursing allows you to get a bird's eye few of how all the gears turn inside the healthcare machine (and it's not always pretty). You may find that nursing is not where you want to stay, put it's a good stepping stone and will pay the bills while you're considering your future options.

Sensei
02-18-12, 01:21
While I have long been aware of this distinction, I have never read it phrased in quite that way. Very clearly cuts right to the heart of one of the critical disconnects which is crippling health care.

So the adage goes: "Build it, and they will come."

I made this realization last time I bought a car. I spent the better part of a day back and forth with the dealer trying to squeeze every bit of value out of my purchase. Once the deal was made, the salesman meticulously introduced me to my vehicle and then gave me his personal cell in case I had any problems or questions once I drove the car off the lot. He even called me a couple days latter to make sure the car was a good fit. Needless to say, a customer service survey arrived in the mail and the large portion of it dealt with my experience with the salesman rather than my opinion of the car.

This got me to thinking about how different my experience as a customer for a new car was when compared to my patients who receive similar surveys. Unlike my patients, who rarely inquire to the price of their care, I spent several hours trying to get the most product for my dollar. Conversely, I can say that I've NEVER had a Medicaid patient ask me about the cost of their visit or the tests that I'll order. Instead, they routinely ask me to perform test or provide care far beyond the scope of an emergency visit ("Hey Doc, while you're at it, write me a script for athletes' foot medicine - Medicaid won't pay for the over the counter stuff..."). Such is the apathy of someone who does not have to pay the bills.

Sadly, I noticed the same problem working at Martin Army Hospital (Ft. Benning, GA) when it came to military dependents. The ED was inundated with dependents who would wait hours for a script for MOTRIN rather than spend the $5 at the PX. That's right, the military was having to pay civilian contractor physicians (~$200/hr) who augmented the active duty docs to handle a "patient" volume that largely needed self-care with Motrin, Tylenol, or Sudafed. Often, they would become indignant when I refused to write the script and instead directed them to isle 17 at the PX. Now, don't get me wrong, I've seen other patient populations risk life and limb for a free bottle of Tylenol, but that was in A-Stan. Ironically, the military also mails our satisfaction surveys, but the bureaucracy has yet to find a means to actually analyze the results.

CDR_Glock
02-18-12, 06:27
Hay guys I'm at a cross roads in my schooling. I'm 3 years out of the mil and have been going to school for a few years now. I want to go into the medical field but I cant decide which diciploine to focus and I need to start applying to programs.

I'm considering Nurse Practitioner or Physician Assistant I know some of you here are EMTs, Medics, Corpsman, PAs and Doctors can you tell me what you think? I want to do more trama related stuff so whats the best choice.

I dont want to go to medical school right now, I'm not even considering it at this point. I live in Rochester NY so there is no shortage of great schools in the area and with the GI bill its a no brainer.

Jon

Thank you for your service.

It depends upon your interest.

For nursing, there are many options. You can be a nurse, educator, administrator, nurse practitioner, or a nurse anesthetist. It seems like a solid pathway. However, Demand varies by region for jobs. As all occupations, you may have to relocate.

As a physician assistant, you have a lot of flexibility, also. You can go into a subspecialty such as internal medicine, surgery, emergency medicine, etc. I perceive with rising costs of healthcare that demand will always be good.

For medicine, as a physician, I don't recommend it in this era. You have 4 years of undergraduate, 4 years of medical school and 3-7 years of residency. You get diminishing returns over time, except for certain specialties such as Anesthesia, Radiology, Dermatology, Plastic Surgery, PM&R, Rad Onc, Cardiology, Gastroenterology, etc. I have done 13 years of education and training but it is one of the most competitive specialties to acquire (during recent time).


iPad/Tapatalk

ICANHITHIMMAN
02-18-12, 06:50
So the adage goes: "Build it, and they will come."

I made this realization last time I bought a car. I spent the better part of a day back and forth with the dealer trying to squeeze every bit of value out of my purchase. Once the deal was made, the salesman meticulously introduced me to my vehicle and then gave me his personal cell in case I had any problems or questions once I drove the car off the lot. He even called me a couple days latter to make sure the car was a good fit. Needless to say, a customer service survey arrived in the mail and the large portion of it dealt with my experience with the salesman rather than my opinion of the car.

This got me to thinking about how different my experience as a customer for a new car was when compared to my patients who receive similar surveys. Unlike my patients, who rarely inquire to the price of their care, I spent several hours trying to get the most product for my dollar. Conversely, I can say that I've NEVER had a Medicaid patient ask me about the cost of their visit or the tests that I'll order. Instead, they routinely ask me to perform test or provide care far beyond the scope of an emergency visit ("Hey Doc, while you're at it, write me a script for athletes' foot medicine - Medicaid won't pay for the over the counter stuff..."). Such is the apathy of someone who does not have to pay the bills.

Sadly, I noticed the same problem working at Martin Army Hospital (Ft. Benning, GA) when it came to military dependents. The ED was inundated with dependents who would wait hours for a script for MOTRIN rather than spend the $5 at the PX. That's right, the military was having to pay civilian contractor physicians (~$200/hr) who augmented the active duty docs to handle a "patient" volume that largely needed self-care with Motrin, Tylenol, or Sudafed. Often, they would become indignant when I refused to write the script and instead directed them to isle 17 at the PX. Now, don't get me wrong, I've seen other patient populations risk life and limb for a free bottle of Tylenol, but that was in A-Stan. Ironically, the military also mails our satisfaction surveys, but the bureaucracy has yet to find a means to actually analyze the results.

I understand your point on just go to the PX thing, but I hope you took into consideration the rank of the dependants spouse when you made these calls. Alot of my friends were on WIC and food stamps until they made E5.

Thanks for the advice its great and I can see how its a double edge sword. I did some time in that hospital you were at in Benning it was a nice place in my opnion, at least I was treated very well.

ICANHITHIMMAN
02-18-12, 06:55
While I have long been aware of this distinction, I have never read it phrased in quite that way. Very clearly cuts right to the heart of one of the critical disconnects which is crippling health care.

To the OP, I found myself in almost exactly your position about 20 years ago when I was confronted with making a mid-life career change (I was not military, however). After completing my EMT training, I had decided to go back to college to get a second degree to allow me to get into health care. My original intention was to complete the necessary pre-requisites to facilitate my acceptance into a PA program (there were far fewer at that time and placement was tight). As I worked my way through the undergraduate healthcare track curriculum (microbiology, A&P, organic chem., etc.), I found myself in more and more classes with nursing students completing their first two years of a four year BSN program. I ultimately decided to get my BSN, get some practical experience, and then make a decision about pursuing a mid-level provider degree. I rationalized that decision by analogizing my RN licensure to a journeyman's license allowing me to find a good-paying job just about anywhere I might find myself. And that same logic holds true today, perhaps even more so.

After spending several years working in the hospital environment, I transitioned into home health care case management. This led me to getting a graduate degree and licensure in the mental health care field. All of this is to say that health care is a BIG area with a wide variety of careers, specialities, work environments, and employment options.

I don't think you will regret getting a BSN. Nursing allows you to get a bird's eye few of how all the gears turn inside the healthcare machine (and it's not always pretty). You may find that nursing is not where you want to stay, put it's a good stepping stone and will pay the bills while you're considering your future options.

I love this first hand experience guys its so helpfull. I can move its not even an option. My wife is in Law enforcement she loves her job and has a 15 years till she can retire. There are alot of jobs in health care where I live. I have already been told by the nursing manager at the local big name hospital that I have a job waitng when I get a degree. We will see if that happens in 2 years.

Sensei
02-18-12, 10:03
I understand your point on just go to the PX thing, but I hope you took into consideration the rank of the dependants spouse when you made these calls. Alot of my friends were on WIC and food stamps until they made E5.

Thanks for the advice its great and I can see how its a double edge sword. I did some time in that hospital you were at in Benning it was a nice place in my opnion, at least I was treated very well.

Understood on the WIC/food stamps situation. These people were not a problem. My frustration was mainly directed at the 21-year old children who were still eligible for TriCare and did not live on-post.

Back to your original question. I would strongly suggest that you look at the Nurse Anesthesia route should you decide on nursing school. Several of my friend have gone that route and LOVE it. Depending on how much your work and where you live, the pay for full time is just north of $100,000 plus benefits. They also have a GREAT lifestyle - in at 6AM and finish most days by 3 PM. Some work nights or take call for more money.

Also, the Army Reserve has a great program for nurse anesthetist. Like physicians, they do 90-day boots-on-the-ground deployments (120-day total) every 2-3 years. You have a significant ability to control the time and location of your deployments if you call them before they call you. You also deploy to theater from the CRC at Ft. Benning which means only a 6 day train-up and a 4 day out-processing. This is MUCH better than the 2-3 months that most reservist spend on the end of each deployment. For example, I did two theater deployments (Iraq 2007 and A-stan 2009) over a 4 year period, then requested a 90-day deployment to Benning when I learned that my wife was prego. This let me commute back every few weeks, while enjoying all of the diversions of my old "home" (combatives, AMU, etc.). You can also make your deployments as high-speed as you want - many who choose to go with a FAST Team earn their CAB or CMB within 30 days of being in theater.

Although they start at 0-2 (I think), most of the nurse anesthetist that I work with made O-4 within 10-12 years since promotions occur consistently within the zone. Send me a PM if you want to learn more, and I'll put you in-touch with people who can explain the nurse anesthesia route a lot better (from both a military and civilian side).

Hmac
02-18-12, 10:28
Back to your original question. I would strongly suggest that you look at the Nurse Anesthesia route should you decide on nursing school. Several of my friend have gone that route and LOVE it. Depending on how much your work and where you live, the pay for full time is just north of $100,000 plus benefits. They also have a GREAT lifestyle - in at 6AM and finish most days by 3 PM. Some work nights or take call for more money.



Agreed, nurse anesthesia is a very cost-effective career path and professionally gratifying, but you have to be interested in the actual work. Being a CRNA is not unlike being an airline pilot... in general, it's hours of boredom interspersed with moments of stark terror. At any rate, that's a decision that can be made as you progress through the educational process. You have to start with a BSN anyway, then a couple of years of critical care experience. A master's degree is universally part of the end result of CRNA school, and many places are awarding a PhD. Even if you get that license, being a CRNA is a great transition to wide variety of other critical care jobs as well as administrative positions.

Pay is generally quite a bit better than being a PA or advanced-practice nurse. The lifestyle is pretty good, although there are call responsibilities. Call is a two-edged sword -- call pay is usually a lot of money, but the lifestyle can take a hit.

Ouroborous
02-18-12, 10:46
Understood on the WIC/food stamps situation. These people were not a problem. My frustration was mainly directed at the 21-year old children who were still eligible for TriCare and did not live on-post.

Back to your original question. I would strongly suggest that you look at the Nurse Anesthesia route should you decide on nursing school. Several of my friend have gone that route and LOVE it. Depending on how much your work and where you live, the pay for full time is just north of $100,000 plus benefits. They also have a GREAT lifestyle - in at 6AM and finish most days by 3 PM. Some work nights or take call for more money.

Also, the Army Reserve has a great program for nurse anesthetist. Like physicians, they do 90-day boots-on-the-ground deployments (120-day total) every 2-3 years. You have a significant ability to control the time and location of your deployments if you call them before they call you. You also deploy to theater from the CRC at Ft. Benning which means only a 6 day train-up and a 4 day out-processing. This is MUCH better than the 2-3 months that most reservist spend on the end of each deployment. For example, I did two theater deployments (Iraq 2007 and A-stan 2009) over a 4 year period, then requested a 90-day deployment to Benning when I learned that my wife was prego. This let me commute back every few weeks, while enjoying all of the diversions of my old "home" (combatives, AMU, etc.). You can also make your deployments as high-speed as you want - many who choose to go with a FAST Team earn their CAB or CMB within 30 days of being in theater.

Although they start at 0-2 (I think), most of the nurse anesthetist that I work with made O-4 within 10-12 years since promotions occur consistently within the zone. Send me a PM if you want to learn more, and I'll put you in-touch with people who can explain the nurse anesthesia route a lot better (from both a military and civilian side).

Not to hijack, but I wouldn't mind learning more about this as well. Like the OP I'm a semester away from applying to an BRN program. Since civilian nursing is so inundated with new grads in my area, my tentative plan is the Navy OCS route with a 3 year tour. What you're talking about sounds like a viable option and one that my family might feel better suits their needs as well.

Hmac
02-18-12, 10:51
Not to hijack, but I wouldn't mind learning more about this as well. Like the OP I'm a semester away from applying to an BRN program. Since civilian nursing is so inundated with new grads in my area, my tentative plan is the Navy OCS route with a 3 year tour. What you're talking about sounds like a viable option and one that my family might feel better suits their needs as well.

One of the CRNA's I work with is ex-.mil CRNA. He speaks very highly of the entire experience.

Ouroborous
02-18-12, 11:31
One of the CRNA's I work with is ex-.mil CRNA. He speaks very highly of the entire experience.

Honestly, going .mil is the thing that's keeping my head in the game. There's just too much competition on the civy side for that first year of required experience. New RN's who refuse to move are waiting 2+ years to get a job- 1000+ applicants for
a single position is the norm nowadays.

Hmac
02-18-12, 11:44
Honestly, going .mil is the thing that's keeping my head in the game. There's just too much competition on the civy side for that first year of required experience. New RN's who refuse to move are waiting 2+ years to get a job- 1000+ applicants for
a single position is the norm nowadays.

Very true. I have a buddy (a CRNA) who has a daughter with a BSN whose end-goal from the beginning was CRNA school. Ultimately she didn't have any trouble getting into CRNA school, but she a) had trouble finding a job as a staff RN at a place with high-level critical care positions (she took a job as a CNA at that hospital for 6 months while waiting for a slot), and b) once she got a job as an RN, had to wait 2 years just to get a job in one of those critical care positions so she could qualify for CRNA school.

montanadave
02-18-12, 11:50
Living in Montana, we have a handful of larger cities with good hospitals and medical centers. However, most of the more rural portions of the state really struggle to staff their smaller hospitals and clinics. Mid-level providers are sought after and the CRNA is a hot ticket.

Obviously, the job market is going to vary but it seems to me, with the obvious need to curtail the escalating spiral in health care costs, mid-level providers are going to be one segment of the personnel spectrum that is sure to expand.

As for going the MD route, it was something I always dreamed about as a kid (I read William Nolan's The Making of a Surgeon when I was about 10 or 12 and thought "Wow! This is the shit!"). However, life took me another direction and I didn't consider healthcare again until I made my mid-life career switch. After working in the acute care setting as a RN and watching the stress, the hours, and the toll on personal and family life good physicians have to pay, I thought on more than one occasion, "Man, did I ever step over a snake. You couldn't pay me enough to live like that."

When making decisions about health care fields, don't limit your selection process to simply areas you find of interest and vocational opportunities. Additional factors to consider carefully include your ability to tolerate and cope effectively with stress, preference for working independently versus working as part of a team, work schedules and balancing those with spouse/family commitments, etc. I learned pretty quick (as did my wife) what areas we were suited to and which ones were likely to put us in an early grave. Don't lock yourself into a speciality area without doing some careful homework and getting some real-world exposure to the actual job environment. There's nothing worse than spending a great deal of time, effort, and money working towards a goal and then realizing, "Shit! What was I thinking?"

I've known a couple of MDs who experienced a very rude awakening when, after dedicating most of their lives towards becoming a physician, making enormous personal sacrifices, and accumulating mountains of debt along the way, they realized they hated the actual job. It's not a pretty sight.

Hmac
02-18-12, 17:34
Medicine isn't for everyone. It's a long, expensive, and stressful educational process, and the job itself, once you finish, is stressful and lifestyle-disruptive. But it's very interesting and very rewarding if you can shrug off the downside. I find it occasionally stressful too, but despite that and the lifestyle impingement, I can't imagine a more rewarding job. The juice is worth the squeeze (for me, anyway), but not everyone sees it the same way. I would never dissuade anyone from going the MD route as long as they knew what they were in for and had the passion to make the sacrifices worthwhile to them and their family.

Sensei
02-19-12, 00:06
Medicine isn't for everyone. It's a long, expensive, and stressful educational process, and the job itself, once you finish, is stressful and lifestyle-disruptive. But it's very interesting and very rewarding if you can shrug off the downside. I find it occasionally stressful too, but despite that and the lifestyle impingement, I can't imagine a more rewarding job. The juice is worth the squeeze (for me, anyway), but not everyone sees it the same way. I would never dissuade anyone from going the MD route as long as they knew what they were in for and had the passion to make the sacrifices worthwhile to them and their family.

I agree. I'd add that one should not be dissuaded by the notion that they are not smart enough or too old to go the MD route. While a certain baseline level of ability is needed to digest the material, a person with a slightly above average IQ can finish the 4 years and get the degree. EQ, or Emotional Equivalent, is just as important as IQ, and is some ways more important for the self-discipline needed to succeed.

In addition, the oldest person in my med school class started at age 48. After med school, he complete a 3-year Family Practice residency and is still practicing 10 years later with no plans of stopping anytime soon...

ICANHITHIMMAN
02-19-12, 08:05
Thanks for the plug on the WIC stuff, I new I was not seeing it from your perspective.

Great stories all of you. I have a freind that is a nurse anesthesia splits his time between the Rochester NY area and Walter Reed he has never said a bad word about it. To be honest though looking at it from the outside in that scares me a bit, I thinks its something I need exposure to firsthand.

usmcvet
02-19-12, 08:15
As a patient I can say I've always been happy with the care I've recived from my PCP but also the group of PA's. My life's been changed/saved by a PA a few times. It sounds like a great return on investment too. Good luck.

Hmac
02-19-12, 08:50
I agree. I'd add that one should not be dissuaded by the notion that they are not smart enough or too old to go the MD route. While a certain baseline level of ability is needed to digest the material, a person with a slightly above average IQ can finish the 4 years and get the degree. EQ, or Emotional Equivalent, is just as important as IQ, and is some ways more important for the self-discipline needed to succeed.

In addition, the oldest person in my med school class started at age 48. After med school, he complete a 3-year Family Practice residency and is still practicing 10 years later with no plans of stopping anytime soon...

I agree, it doesn't take extraordinary intelligence. It does take commitment, self-discipline, and the ability to memorize large quantities of information.

As to age..I have a brother-in-law who practiced for 5 years, but for the last 20 has held various executive positions with various insurance companies and senior-care organizations. OTOH, I have another BIL (his brother) that was a computer systems analyst, then at age 45 decided to go to medical school. He finished his psychiatry residency at age 51. Medical schools are more and more looking toward what they call the "non-traditional" student. They are finding great success in the people who have been out in the world and are a little older. Still have to have kick-ass grades and MCAT scores, but many schools like the level of maturity that they don't necessarily see in the medical students that come in right out of college. One of the big reasons for that, I'm told by the Dean of Admissions, is that those few years out in the world tend to help people to lose the sense of entitlement they may have had up through college. Medical school can be academically brutal. There is no grade inflation at that level education and those that moan about unfair grading are pretty much just ignored.

Hmac
02-25-12, 07:14
I thought this article from Medscape was interesting. It's a report on a paper that was presented at the American Academy of Orthopedic Surgeons most recent annual meeting a couple of weeks ago. As I have suspected, Nurse Anesthesia remains one of the most cost-effective medical career choices. Registered Nurse/Nurse Practitioner...not so much. As for going to medical school and becoming a doctor...bad career investment (unless you really love it). And that's if starting the path out of high school. The later in life you start, the worse that choice is from a career investment standpoint.

(Note that the article is not copyrighted, and terms of my subscription allows reprinting)



February 15, 2012 (San Francisco, California) — Orthopedic surgeons are well paid, but the profession is less lucrative than some other career choices, making orthopedics training a "poor financial investment," researchers reported here at the American Academy of Orthopaedic Surgeons 2012 Annual Meeting.

"If this continues over time, we may lose qualified individuals in our field, and this may reduce access to good quality orthopedic care, orthopedic research, and orthopedic management," warned Suneel B. Bhat, MD, an orthopedic surgery resident at Thomas Jefferson University Hospital in Philadelphia, Pennsylvania.

To see how orthopedic surgery stacks up as a career, Dr. Bhat and his colleagues compared orthopedic surgeons to dentists, lawyers, certified registered nurse anesthetists, and nurse practitioners.

"We simulate an individual entering a profession at age 18 or 19, taking on loans, going through school, emerging, starting a salary, paying back their loans, and then saving some every year once they have a stable income," said Dr. Bhat.

Using various surveys from professional organizations, the team estimated after-tax income, savings and interest, and liabilities such as student loans. The medical profession income data were from 2008 and 2009, but the law profession data were from 1997.

They assumed that educational loans would be deferred until the annual liability was less than 25% of earnings, and that interest on the loans was 8.25%.

They also assumed that the individual would save 0.15% of earnings after achieving a stable income, and that investments paid an interest rate of 0.5%.

Looking at lifetime cumulative earnings, they found that orthopedic surgeons do better than the other 4 professions, reaching $10,756,190 by age 65.

In comparison, lawyers earned a cumulative $8,381,250, certified registered nurse anesthetists $7,338,412, dentists $6,866,796, and nurse practitioners $3,867,504.

But orthopedic surgeons, it seems, work longer hours. When the researchers calculated hourly income for the 5 professions, lawyers came out on top, at $130; dentists were next at $99, followed by nurse anesthetists at $93. Orthopedic surgeons came in fourth, with a relatively modest $88, although still well ahead of nurse practitioners, who made $49.

"In absolute terms, orthopedic surgeons do well," said Dr. Bhat. "However, when you adjust for hours of work, orthopedics is not a very good financial investment. In fact, you would almost be better to go into another field and work as hard."

He identified several trends that might be eroding orthopedic surgeons' incomes. Since 1984, medical school tuition in public institutions has risen 165% and in private schools has risen 312%.

In the past 5 years, the mean debts of orthopedic surgery students have increased from $86,000 to more than $120,000 in public schools and from $120,000 to more than $160,000 in private schools.

Meanwhile, decreases in Medicare fee schedules and the increased market penetration of managed care has caused orthopedic surgeons' salaries to stagnate.

Asked during a panel discussion whether he would choose a different profession if he had the decision to make over again, Dr. Bhat acknowledged that he would still choose orthopedic surgery. "But I can't help but look at my buddies in college," he said; "they had the same offers, and they went the other way."

Session moderator Joseph Dias, MD, professor of orthopedic surgery at the University of Leicester, United Kingdom, and president of the British Orthopaedic Association, speculated that most orthopedic surgeons would say the same. "I think people go into medicine mainly because they have seen how much doctors enjoy being doctors," he told Medscape Medical News.

"Often they have a glorified view, but after they start, they see that most of the time we are relieving people of their pain. We help people.... So we get engaged in what we do."

Dr. Bhat and Dr. Dias have disclosed no relevant financial relationships.

American Academy of Orthopaedic Surgeons (AAOS) 2012 Annual Meeting: Abstract 746. Presented February 10, 2012.

FL2011
02-28-12, 22:15
I'm a medical resident so and would recommend the MD route with some heavy hesitation. Certainly not for everyone and prob not the best option in regards to the payout compared to time and energy commitment required.

That said I love the field of medicine and while there are days I feel like walking away I can't imagine a more interesting and rewarding career.

I also seriously considered PA school originally and have friends who went that route and are extremely happy. Plus they have the ability to move among different specialities easier than I will after completing a residency/fellowship.

I work with NPs and PAs daily and the ones we have are terrific. At least in the in-patient setting where I'm at they pretty much have similar roles. Overall I think PAs are more common on the surgical services, but even then we have a cardio thoracic surgery NP who is terrific.

Whatever you do put a LOT of thought into the financial aspect of keeping loans and cost down. Maybe I'm just a little bitter but I just signed a mortgage that's less than my med school loan debt........

TacMedic556
03-01-12, 20:50
I attended a Fire Academy, EMT-B course at 19. Became a Paramedic after a 2 year AAS course by 22. After 10 years as a full time firefighter paramedic, I can tell you it is the best and most rewarding job on earth (for me IMO). 6 years ago I applied and got on SWAT as the only medic. It is the best of all worlds.

Get your medic. Apply and make a career of it. I recommend it and recruit guys as much as I can.

ICANHITHIMMAN
03-02-12, 14:58
I attended a Fire Academy, EMT-B course at 19. Became a Paramedic after a 2 year AAS course by 22. After 10 years as a full time firefighter paramedic, I can tell you it is the best and most rewarding job on earth (for me IMO). 6 years ago I applied and got on SWAT as the only medic. It is the best of all worlds.

Get your medic. Apply and make a career of it. I recommend it and recruit guys as much as I can.

Thanks for the advice, but Im a 32 year old white combat vet which all seems to be held against me. I will never get one of thouse jobs where I live unless Im related. It sounds like a super cool job you have to say the least. What type of stuff do you see on a day to day.

Texas42
03-02-12, 16:31
Thanks for the advice, but Im a 32 year old white combat vet which all seems to be held against me. I will never get one of thouse jobs where I live unless Im related. It sounds like a super cool job you have to say the least. What type of stuff do you see on a day to day.

Lots of people in my medical school class were in their late 30's. Some evel looked early 40's.

Just saying.

ICANHITHIMMAN
03-02-12, 16:34
Lots of people in my medical school class were in their late 30's. Some evel looked early 40's.

Just saying.

HAHA I dont think I'm old I was just saying but thanks for the heads up. I guess I was just venting on that job subject.

TacMedic556
03-02-12, 17:23
I've known many vets and white guys in their 30's that successfully applied and were hired, even attended the schooling while married with kids. Here in Montana you get veterans POINTS added to your testing score which helps place you higher on the hiring lists.

Give it a shot if it is something you would like to do. You can do it.

ICANHITHIMMAN
03-02-12, 19:21
I have tried 5 times it takes a year every time. The local department where I live has a policy 3 out of every 4 they hire has to be a minority. That’s not a lot of room for people who are not connected. I also cannot move so a state job is out of the question as well as most of the county’s, as you have to live in them.

The local doc who does the psych is also not friendly to vets he thinks we are all mental and there is a pending discrimination suit against him.

Tact.medic
03-20-12, 13:02
Hay guys I'm at a cross roads in my schooling. I'm 3 years out of the mil and have been going to school for a few years now. I want to go into the medical field but I cant decide which diciploine to focus and I need to start applying to programs.

I'm considering Nurse Practitioner or Physician Assistant I know some of you here are EMTs, Medics, Corpsman, PAs and Doctors can you tell me what you think? I want to do more trama related stuff so whats the best choice.

I dont want to go to medical school right now, I'm not even considering it at this point. I live in Rochester NY so there is no shortage of great schools in the area and with the GI bill its a no brainer.

Jon

I say go to be a Nurse practitioner or PA. I was a EMT-P in NYC, and it is frustrating to get paid beans for theamount of trauma and real medical emergencies you get PER DAY. But the real issue is the amount of abuse your body and mind take, there is a lot of lifting (try carrying a 270lb 6'4" male down 4 floors along with your gear, and doing CPR). Do not use the "medic as a stepping stone" ideallogy since it is crap- Stick to career and stay on that education pathway, you will reap the benifits in the long run :big_boss:

TacMedic556
03-20-12, 15:11
Here in the NW US we FF / EMT - P start anywhere from $45K-$60K a year. Benefits are great. Most schedules are 24h on with 48 off with an occasional Kelly day (dropped shift to lower hours worked per FLSA). This coupled with your vacation leave dates generates considerable options for time off at times. The days can be long but camaraderie is great. You cannot beat the job. Best job there is IMO. Ten years on, hope to see 10 -15 more.

Tact.medic
03-21-12, 06:51
I have tried 5 times it takes a year every time. The local department where I live has a policy 3 out of every 4 they hire has to be a minority. That’s not a lot of room for people who are not connected. I also cannot move so a state job is out of the question as well as most of the county’s, as you have to live in them.

The local doc who does the psych is also not friendly to vets he thinks we are all mental and there is a pending discrimination suit against him.

LOL, for real? I have to throw the BS flag up, 3 out every 4? where do you live montgomery alabama? I know live in Orlando and work in a primarily white fire dept (I'm a FF/PM) and I can tell you that even though we are mostly a white dept we do hire only the best qualified applicants. Its just that most of them are white, you see?

I agree that being a FF/PM with a dept that has K-Days and a 24-48 schedule is great. But this is not for everyone, as I said if you want the best bang for the buch then go become an RN and work in the truama room. You will have great pay, and the only person you have to really deal with is the Doc

ICANHITHIMMAN
03-21-12, 10:08
LOL, for real? I have to throw the BS flag up, 3 out every 4? where do you live montgomery alabama? I know live in Orlando and work in a primarily white fire dept (I'm a FF/PM) and I can tell you that even though we are mostly a white dept we do hire only the best qualified applicants. Its just that most of them are white, you see?

I agree that being a FF/PM with a dept that has K-Days and a 24-48 schedule is great. But this is not for everyone, as I said if you want the best bang for the buch then go become an RN and work in the truama room. You will have great pay, and the only person you have to really deal with is the Doc

I live around Rochester NY and I wish it was BS

ICANHITHIMMAN
03-21-12, 10:10
One math class this summer and I will have my 2 year degree in LA heath related. Then one more semester of pre reqs, Micro, Nutrition, Organic chem and I can transfes as a junior right into the nursing program for my BS.

Shoulderthinggoesup
03-21-12, 10:58
EMT-B here. Going into Duke's PA program starting in the fall. Obviously I am going to second the PA/NP option. PA is 2 years to a good salary practicing medicine.

Sent from my PC36100 using Tapatalk

Ouroborous
03-22-12, 22:46
One math class this summer and I will have my 2 year degree in LA heath related. Then one more semester of pre reqs, Micro, Nutrition, Organic chem and I can transfes as a junior right into the nursing program for my BS.

Congrats on being a semester away from having your pre-req's done. I've got physio and micro to do and I'm done as well.

Consider challenging the paramedic exam after you become an RN to boost your cred's for the fireservice. I understand what you're saying but getting your medic will help a lot. Those veterans preference points help too brother.

Either way you'll be able to work and if your local dept's aren't hiring, test elsewhere-you can make it happen if that's what you really want to do.

Ouroborous
03-22-12, 23:02
EMT-B here. Going into Duke's PA program starting in the fall. Obviously I am going to second the PA/NP option. PA is 2 years to a good salary practicing medicine.

Sent from my PC36100 using Tapatalk


What was the application process like for your PA school? Around here it is a lottery with the best gpa's getting first dibs. One program saw 1200+ applicants for fall 2011 and only had space for 40.
Wish I had more time and money for school else i'd go that route too.

ICANHITHIMMAN
03-23-12, 05:30
What was the application process like for your PA school? Around here it is a lottery with the best gpa's getting first dibs. One program saw 1200+ applicants for fall 2011 and only had space for 40.
Wish I had more time and money for school else i'd go that route too.

Thats how RIT is

Reagans Rascals
03-23-12, 05:54
I'd concentrate on getting my RN first.... if you have no desire to pursue an MD... I'd go for Nurse Anesthetist later down the road... from what I have heard, its probably one of the highest paid nursing positions you can attain....

browningboy84
04-02-12, 20:29
CRNA is an excellent way to go. That is my end goal when I decide to quit working offshore. Get your RN, and work your way into a ICU/CCU/SICU. Put your time in there, get your BSN, and then start applying. The money is definitely better than what a PA or a Nurse Practitioner makes.

Shoulderthinggoesup
04-03-12, 01:46
What was the application process like for your PA school? Around here it is a lottery with the best gpa's getting first dibs. One program saw 1200+ applicants for fall 2011 and only had space for 40.
Wish I had more time and money for school else i'd go that route too.

It is pretty damn competitive, with most schools getting between 800-2000 applications for 24-70 spots. I had a 3.96+ GPA (last 128 Hours..a few hiccups before that) /1440 GRE and a decent amount of health care experience.

I applied to 10 schools, got interviews a 5, got accepted to 3 and waitlisted at 1.

It sucked, but at least I don't have do it all over again for residencies like the med students do.



On a side note, CRNA is a cool route if anesthesia is up your alley and you are ALREADY a nurse.... but if your starting from scratch just do med school....

ICANHITHIMMAN
04-03-12, 09:25
Oh my freind Chris just passed his board for that.

ICANHITHIMMAN
11-06-12, 10:01
OK I need some advise, I am failing organic chemistry. I know the stuff cold but when it comes to the exams i'm getting in the 60%. I have sat down with the professor after ever exam, to try and figure it out and it boils down to her questions suck! I use a tutor and help others no one seams to be able to help me. What can I do? I'm very depressed about this, I have tried a bunch of tactics and its not working. Its killing my GPA and I cant drop or I loose my full time GI bill.

WS6
11-06-12, 10:16
OK I need some advise, I am failing organic chemistry. I know the stuff cold but when it comes to the exams i'm getting in the 60%. I have sat down with the professor after ever exam, to try and figure it out and it boils down to her questions suck! I use a tutor and help others no one seams to be able to help me. What can I do? I'm very depressed about this, I have tried a bunch of tactics and its not working. Its killing my GPA and I cant drop or I loose my full time GI bill.

Btdt, man. Sometimes you just get a professor that you can't deal with on tests. I don't know what to say other than try your ass off. Find someone who "gets" that professor and pay them to drill you on the material if you have to. My courses, you only got 4-6 tests at the very most, the 4th one or whatever was last, being the final. It was cumulative, and worth 30% or so, usually. It is not where you want to have to make up your previous fails. If you already have more than one sixtys fail, I'd say you're dead. Anything less than an 80 was a fail, effectively, in my program. I will say this, though, to encourage you. Never! Give up! I passed medsurg by literally one question. I was timed out on the final, and before the instructor physically room my test (I was the last in the room, all others had left, and I was staying until the end--I was trying to make it up on the cumulative final.) I asked one question. It was about a barium swallow imaging test. They told me "well you need to know everything about that test if you're going to get this right." And moved to take my finished test. I quickly erased and changed my answer, smiled, and said "we will see if that was smart of me or not" and walked out. That question was correctly answered. Saved my semester and maybe career.

kiwi57
11-06-12, 11:31
ICANHITHIMMAN,

Organic, especially when it is taught from an ivory tower perspective, can be really frustrating. An instructor who will constantly emphasize the clinical applications of the concepts is a rare bird indeed. You might want to check with the student support center (or learning center--whatever your institution calls it) and see if they can help you with testing strategies. Sometimes we can be our own worst enemy when it comes to test taking habits. Also, they might be able to help with study strategies that might benefit your performance. Usually these folks have a solid education background and can be of help. Sometimes not, but it may be worth a try.

Kiwi57

ICANHITHIMMAN
11-06-12, 13:41
I have at least 4 more exams they are 60% of my grade. The labs and quiz comprise the other 40%. Worst case is I take it again but I hate to do that, just hate it.

WS6
11-06-12, 15:09
I have at least 4 more exams they are 60% of my grade. The labs and quiz comprise the other 40%. Worst case is I take it again but I hate to do that, just hate it.

The above poster is right---change your approach. I was doing very bad studying powerpoints and notes. I stopped all of that and all I did was read my text-book. I went from doing poorly, to making high 90's on my tests in one class, just by doing that. I'm not saying that is how I would treat Organic, but I'm just saying that you're not getting it done how you're doing now, so try another way.

ICANHITHIMMAN
11-06-12, 16:23
The above poster is right---change your approach. I was doing very bad studying powerpoints and notes. I stopped all of that and all I did was read my text-book. I went from doing poorly, to making high 90's on my tests in one class, just by doing that. I'm not saying that is how I would treat Organic, but I'm just saying that you're not getting it done how you're doing now, so try another way.

No I agree, I'm reading the chapters, doing flash cards and all the sample problems in each chapter. I use a tutor in the science learning center and visit the professor during office hours. She knows I know the stuff but I cant pass the exams. I'm super frustrated, each time I have sat down with her and gone over the exam. We talk about each question and I get it right but the way she ask's it in here written questions I get it wrong. There are about 21 questions on each exam.

Royalflush
12-04-12, 15:12
organic blows man- but it's something we all have to take!

back in the day I took it without any understanding of the general chemistry I had taken prior to organic. I had just taken gen chem to pass the tests. after I flunked my first orgo exam, i went back to the gen chem books and re-learned the basics.

Using 3-d models seemed to help me visualize the structures quite a bit better. I did a biology and chemistry degrees, I've got friends in med school and PA school- I'm in vet med school and trust me, you aren't going to really need to use what you are doing. I'm doing really well in pharmacology and I don't remember crap from organic- biochemistry is much more important for the field of study that you are planning on going into.

on that note: PA is a great option for you! you'll always have a job and there is alot of variety in that field. MD is declining as a profession, it's not going to be like it was in the past few decades. The more gov. intervention in human med=less money and more work for doctors.
PA's will be the new doctor in most outpatient clinics in a decade. there will always be MDs, but I'm thinking there will be 5 PA's in a clinic per MD as opposed to the 1:1 or 2:1 ratio now.

WillBrink
12-04-12, 16:03
I have at least 4 more exams they are 60% of my grade. The labs and quiz comprise the other 40%. Worst case is I take it again but I hate to do that, just hate it.

Lots of people have had to re take organic chem. Some schools only allow people to re take it 3 times. Some need all 3. Being organic has essentially no math, I did OK with it. I'll say this, then give you #1 thing that worked for me. Just be happy you don't have to take bio chem, because you'd be praying for good old organic chem test in the middle of a bio chem test. 2 semesters of general chem, two semesters of organic were 'easy' compared to bio chem, which was a real mother fu$#@#er I tell you. And the mere thought of the course I took after bio chem called Biochemical and Molecular Physiology makes me shed hair to this day.

OK, besides the usual stuff (study, study, and study, own a 3D model kit for yourself, flash cards, tutor, study groups, study, take good notes, study, etc) here's one trick that really saved my ass:

Record every class. I recorded ever class, and would then play it back in my car, on my head phones while on the treadmill, and in the library and take additional notes at my own pace.

That did two primary things: it allowed me to really pay attention to the instructor without worrying about missing some essential point, and the extreme repetition of hearing it over and over really drilled it into my memory banks.

So before proff started, i set up my little recorder and pushed record for every damn class. That was pre electronic media so I had stacks of cassette tapes on my desk. Today I'd probably video the class and both watch and listen to it and take notes as vid is so cheap.

Best thing I ever did.

I never missed a class (as in never ever short of end of the world), sat in front row (that matters more than you may think), used proffs office times so he/she knew I was very serious about "getting it" went to study group every week, and lived, slept, ate bio chemistry 'cause come hell or high water I was not taking that mother f-er again.

ICANHITHIMMAN
12-04-12, 17:26
Thanks to the both of you for your comments. I have never missed a class, I sit in the front row and I am focused. I understand the material and I generally enjoy it. The professor is great and she goes above and beyond to help. I have read all the chapters and performed all of the problems in the book(and gotten them correct). Even did the back tracking think mentioned.

My problem is the test, I flat out SUCK at taking them, it has been that way my entire life. I could teach it to you in the morning and fail the test that afternoon while you got 100%. Dont laugh it has happened more than once. Math is the same way and I understand it, just always seam to bomb the exams. First 4 exams in organic were mostly math.

All professors have office hours and between there and the different study centers, I am all out of options for this semester. I have to retake it now, but I'm not freaking out or anything. I just started with a math tutor and I am hoping to hell it corrects shit. The professors don't know how to help me, cause I know the stuff and they know it, I have proven it to them. I actually went to my doctor yesterday to get a referral to be tested for a learning disability so that I can, maybe if they can identify something I can correct it. I have been in school 2 years now and I should be transferring to my 4 year but I need another math class to complete my degree. Its very frustrating, my doctor actual laughed at me and said he hated that class it was hard lol WTF

WillBrink
12-04-12, 18:26
Thanks to the both of you for your comments. I have never missed a class, I sit in the front row and I am focused. I understand the material and I generally enjoy it. The professor is great and she goes above and beyond to help. I have read all the chapters and performed all of the problems in the book(and gotten them correct). Even did the back tracking think mentioned.

My problem is the test, I flat out SUCK at taking them, it has been that way my entire life. I could teach it to you in the morning and fail the test that afternoon while you got 100%. Dont laugh it has happened more than once. Math is the same way and I understand it, just always seam to bomb the exams. First 4 exams in organic were mostly math.

All professors have office hours and between there and the different study centers, I am all out of options for this semester. I have to retake it now, but I'm not freaking out or anything. I just started with a math tutor and I am hoping to hell it corrects shit. The professors don't know how to help me, cause I know the stuff and they know it, I have proven it to them. I actually went to my doctor yesterday to get a referral to be tested for a learning disability so that I can, maybe if they can identify something I can correct it. I have been in school 2 years now and I should be transferring to my 4 year but I need another math class to complete my degree. Its very frustrating, my doctor actual laughed at me and said he hated that class it was hard lol WTF

I'm not a good test taker either. Next time record the class as i said above. Makes a big difference in my experience and i was a professional student for many years. ;)

ICANHITHIMMAN
12-04-12, 18:47
thanks Will

Royalflush
12-04-12, 19:06
will,
you and I had completely different org/biochem experiences! I hated organic- had trouble with it and waltz right through biochem because it just clicked!
Same thing with calculus: had trouble with it- couldn't get a anything above a C- (and I was trying!) , but aced every other upper level math, stats and physics class I took....
goes to show peoples minds are truly geared different, I think alot of that has to do with the way you visualize things

Ouroborous
12-04-12, 20:55
My problem is the test, I flat out SUCK at taking them, it has been that way my entire life. I could teach it to you in the morning and fail the test that afternoon while you got 100%. Dont laugh it has happened more than once. Math is the same way and I understand it, just always seam to bomb the exams. First 4 exams in organic were mostly math.

All professors have office hours and between there and the different study centers, I am all out of options for this semester. I have to retake it now, but I'm not freaking out or anything. I just started with a math tutor and I am hoping to hell it corrects shit. The professors don't know how to help me, cause I know the stuff and they know it, I have proven it to them. I actually went to my doctor yesterday to get a referral to be tested for a learning disability so that I can, maybe if they can identify something I can correct it. I have been in school 2 years now and I should be transferring to my 4 year but I need another math class to complete my degree. Its very frustrating, my doctor actual laughed at me and said he hated that class it was hard lol WTF

In hopes of helping you-at the very least to let you know your situation is not unique- here's my personal experience:

I hear you on test taking. I'm finishing my last RN pre-req (Physio). Throughout my 3 year career as a student, I have consistently been outdone by people I help with the material.

Example:

During my last exam's study session, my partner was soo swamped with Organic Chem that she didn't have much time to study for the exam. We spent two hours on the material prior to the exam going through my flash cards and the book. When we got our test results back, she got 106 (A+) and I got 87 (B). She claimed there was no way she would have done as well as she had without my help that morning.

Unfortunately, this isn't some isolated occurrence as this has happened in literally every heady subject I've ever taken-I tutor people to help myself learn and they seem to do better than me on exams. I'm simply better at explaining material than taking tests. Although there are other ways of assessing knowledge and skill, the entire academic construct is geared for people who take tests well.

I have heard of teachers offering verbal exams where you explain the material to the teacher during a one on one meeting-I suspect people like you and I would do better in these situations if that were available.

I never bothered getting tested for a learning disability because I (wrongfully) assumed that having test anxiety meant some kind of debilitating condition where you fall apart at the seams when faced with exams- I never fell apart when going on a trauma call or a rescue why should I not be able take a simple test?

It wasn't until becoming a tutor this semester that I learned that common symptoms of mild test anxiety are inability to comprehend exam questions and "blanking" or "brain farts"-the same situations I've experienced over the past 3 years.

If you can be assessed for a learning disability do it. If they find that you have this then extra time and private testing areas are made available and this will lend greatly to your success as a student. In the meantime hang in there and know there's probably more than a few people who're going through the same thing including myself.

ICANHITHIMMAN
12-05-12, 06:59
will,
you and I had completely different org/biochem experiences! I hated organic- had trouble with it and waltz right through biochem because it just clicked!
Same thing with calculus: had trouble with it- couldn't get a anything above a C- (and I was trying!) , but aced every other upper level math, stats and physics class I took....
goes to show peoples minds are truly geared different, I think alot of that has to do with the way you visualize things

Its funny about the math I mess up the simple stuff on the test but get the complex stuff correct

ICANHITHIMMAN
12-05-12, 07:06
In hopes of helping you-at the very least to let you know your situation is not unique- here's my personal experience:

I hear you on test taking. I'm finishing my last RN pre-req (Physio). Throughout my 3 year career as a student, I have consistently been outdone by people I help with the material.

Example:

During my last exam's study session, my partner was soo swamped with Organic Chem that she didn't have much time to study for the exam. We spent two hours on the material prior to the exam going through my flash cards and the book. When we got our test results back, she got 106 (A+) and I got 87 (B). She claimed there was no way she would have done as well as she had without my help that morning.

Unfortunately, this isn't some isolated occurrence as this has happened in literally every heady subject I've ever taken-I tutor people to help myself learn and they seem to do better than me on exams. I'm simply better at explaining material than taking tests. Although there are other ways of assessing knowledge and skill, the entire academic construct is geared for people who take tests well.

I have heard of teachers offering verbal exams where you explain the material to the teacher during a one on one meeting-I suspect people like you and I would do better in these situations if that were available.

I never bothered getting tested for a learning disability because I (wrongfully) assumed that having test anxiety meant some kind of debilitating condition where you fall apart at the seams when faced with exams- I never fell apart when going on a trauma call or a rescue why should I not be able take a simple test?

It wasn't until becoming a tutor this semester that I learned that common symptoms of mild test anxiety are inability to comprehend exam questions and "blanking" or "brain farts"-the same situations I've experienced over the past 3 years.

If you can be assessed for a learning disability do it. If they find that you have this then extra time and private testing areas are made available and this will lend greatly to your success as a student. In the meantime hang in there and know there's probably more than a few people who're going through the same thing including myself.
Thanks we have the same goal, I took phys already great glass, I have to get organic, micro and stats 1 out of the way still, taking nutrition just as a filler, plus they let me drop some transfer credits to get my full time and retake Arabic (I already went to the SOLT school) so its cake and keeps my GPA up. Thing is I don't feel stressed I feel confident, its very defeating coming from 9 years of being the man to not passing a simple exam. But I'm going to get it done

I have been thinking about the PA route because RIT has a great program and my buddy wants to do it to. He already applied but got shipped out on orders, he has a shit tone or pre reqs to do still so I'm not sure how he did that. I don't know what kind of credits he had from flight school.

WillBrink
12-05-12, 07:48
will,
you and I had completely different org/biochem experiences! I hated organic- had trouble with it and waltz right through biochem because it just clicked!
Same thing with calculus: had trouble with it- couldn't get a anything above a C- (and I was trying!) , but aced every other upper level math, stats and physics class I took....
goes to show peoples minds are truly geared different, I think alot of that has to do with the way you visualize things

Actually, I didn't really enjoy organic per se, and did OK at it. Bio chem much more complex and difficult course I found, but and the only reason I did OK at it (B+ I recall) was because it was so interesting to me. That's what saved me really. All the chem before bio comes together at bio chem and get into the interesting stuff you waited to prior courses for. Had bio chem not been interesting and useful (unlike organic!) I don't think I would have survived it.

Ouroborous
12-05-12, 11:42
I had to take an Allied health Chem class that included inorganic, organic, and biochem. None of it went into the same amount of depth as the actual class since it was only a semester.

I scrapped by with C's and D's during the inorganic and it finally started clicking for me during the organic and bio portions. For most people in the class it was the complete opposite. Made it out out with a B.

Just goes to show how some people's brains do better with certain subjects than others.

Will, from what I experienced Biochem was definitely more practical and interesting compared to the other portions of the class. Did you have to delve into all the metabolic pathways?

jmnielsen
12-05-12, 11:56
I scrapped by with C's and D's during the inorganic and it finally started clicking for me during the organic and bio portions. For most people in the class it was the complete opposite. Made it out out with a B.


I'm hoping that's my case. General chem has been whooping me. Inhabe heard from a lot of people where I go to school though that organic is easier than inorganic- guess ill find out next semester.

WillBrink
12-05-12, 17:08
I had to take an Allied health Chem class that included inorganic, organic, and biochem. None of it went into the same amount of depth as the actual class since it was only a semester.

I scrapped by with C's and D's during the inorganic and it finally started clicking for me during the organic and bio portions. For most people in the class it was the complete opposite. Made it out out with a B.

Just goes to show how some people's brains do better with certain subjects than others.

Will, from what I experienced Biochem was definitely more practical and interesting compared to the other portions of the class. Did you have to delve into all the metabolic pathways?

You bet, had to memorize the entire glycolytic pathway, TCA cycle, all products, reactions, side reactions, ATPs gained and lost, NADPH/NDAP+ comings and goings, etc and others I recall fondly. And by fondly, I mean I lost a lot of hair that two semesters I recall. :meeting:

Bio chem is much more interesting and applied to what I had actual interest in (human bio) but it was worst of them all to me in terms of memorization, formulas, pathways, etc, etc.

Had I not found it so interesting and useful and important to my overall goals, I wouldn't have survived it I think. There were many in my class taking it for the second time, and a few taking it for the third! Three times was max allowed.

The proff made it clear that as far as he was concerned bio chem was designed to "separate the wheat from the chaff" and if you couldn't get past it you had no business going to med school, which was where most were going after the finished their pre med sciences.

WillBrink
12-05-12, 17:12
I'm hoping that's my case. General chem has been whooping me. Inhabe heard from a lot of people where I go to school though that organic is easier than inorganic- guess ill find out next semester.

Depends on how your brain works. Organic has minimal math, so if the math portion & formulas in General (I recall the gas constant formulas were a PITA for me for example) are what's messing you up, Org is easier. But, some people wiz through the math and have trouble with the concepts and are not able to deal with the three dimensional physical aspects of organic, and crash and burn on it.

Bio chem, they all come together to kick your ass in my experience, even though it's the most interesting of them by far, which helps a lot!

Texas42
12-05-12, 18:14
Depends on how your brain works. Organic has minimal math, so if the math portion & formulas in General (I recall the gas constant formulas were a PITA for me for example) are what's messing you up, Org is easier. But, some people wiz through the math and have trouble with the concepts and are not able to deal with the three dimensional physical aspects of organic, and crash and burn on it.

Bio chem, they all come together to kick your ass in my experience, even though it's the most interesting of them by far, which helps a lot!

Depends on your professor.

I actually enjoyed most of my organic chemisty (well. . . looking back). The lab was fun. I actually did a lot better in organic chem than biochem. . . My degree is in Biochem, but I really don't remember a whole lot. Though biochem was a breese in medical school. My inorganic chemistry was rough, but very managable. Much of the class didn't do well, but I tutored people in another professor's class. . . . that professor was an asswhole. Throwing tons of organic in his gen chem class. Just took a way of looking at and understanding things.

WillBrink
12-05-12, 18:21
Depends on your professor.


Very much so! First attempt at bio chem I withdrew from the class in the first few weeks as I disliked the teacher from the start. Ended up taking it at a different school.

Hmac
12-05-12, 18:25
My degree is in Biochem, but I really don't remember a whole lot. Though biochem was a breese in medical school. My inorganic chemistry was rough, but very managable.

I didn't take biochemistry in college, but I did find it to be pretty straightforward in medical school. I suspect the difference is that in medical school, no one was gunning for grades in order to get into medical school. The competition from all the premed nerds and therefore the grading curve for all the chemistry courses in college was brutal.

/

Texas42
12-05-12, 18:42
I still have my biochem, org chem, and gen chem textbook. . you know, just in case I want to re-learn all that I forgot. :)

WillBrink
12-05-12, 19:05
I didn't take biochemistry in college, but I did find it to be pretty straightforward in medical school. I suspect the difference is that in medical school, no one was gunning for grades in order to get into medical school. The competition from all the premed nerds and therefore the grading curve for all the chemistry courses in college was brutal.

/

They were major gunning for grades in my huge lecture class and probably 90& of them applying to Ivy League med schools. My study partner was accepted into Harvard Med and got 100 on all tests in the class. Talk about f-ing up the grade curve. He was a smart SOB I tell you, but couldn't tie his own shoes half the time. Me, I was not gunning, but surviving and was thrilled with a B+ for the course.

ICANHITHIMMAN
12-05-12, 19:31
This is good to know thanks guys. Worst part is the ****ing cheaters, I notice everything! Its a direct result of INSPECTIONS, and looking for stuff that wants to hurt you overseas.

I bet I have seen 5 people in my organic class alone using there cell phones to cheat.

Hmac
12-05-12, 19:31
The proff made it clear that as far as he was concerned bio chem was designed to "separate the wheat from the chaff" and if you couldn't get past it you had no business going to med school, which was where most were going after the finished their pre med sciences.

Relatively few medical schools require college biochemistry for admission, but 2 years of chemistry (inorganic and organic) have been standard requirements for decades. And for almost the entire history of modern premedical education, Organic Chemistry has been the stumbling block...the course where professors feel obliged to separate out the wheat from the chaff, and probably the science course that medical school admissions committees look at grades most closely in order to evaluate an applicant for admission.

fdny2pa
12-06-12, 14:19
If you look at my SN I am advising PA school. I'm former USMC/FDNY now a PA working in Internal Medicine on Long Island NY. I think its an awesome career choice. Message me if there are any questions I can answer for you

Shoulderthinggoesup
12-07-12, 22:44
Inorganic was a real PITA. I personally found Organic Chem to be a lot of fun though! Puzzles!

ICANHITHIMMAN
12-08-12, 05:33
If you look at my SN I am advising PA school. I'm former USMC/FDNY now a PA working in Internal Medicine on Long Island NY. I think its an awesome career choice. Message me if there are any questions I can answer for you

That's kind of you, if you lived a little closer it would be great to head to work with you for a day. I wont say I haven't considered it, just haven't explored it as in depth. You do need a bachelors first right?

ICANHITHIMMAN
12-08-12, 05:41
Inorganic was a real PITA. I personally found Organic Chem to be a lot of fun though! Puzzles!

I agree its a great glass, I love learning how everything is connected. The only issue for me with all these classes seams to be every time I learn about some new disease I think i have it.:D.

Shoulderthinggoesup
12-08-12, 11:11
lol, your gonna have an aneurysm when you get into clinical medicine classes.....

In all seriousness though, PA is a great route (though I am biased, as I am in PA school @ duke right now). You get to practice medicine without giving up a decade of your life to do so and get pretty decent pay to boot. The school itself is absolutely insane, but we seem to be making it through. I am stalling on studying for my ortho exam right at this second in fact. Really nothing as far as concepts is as hard as Chem in PA school, but the load of information is similar to taking 3-4 four week summer science classes every month. Tests on 1200 slides with 2-3 days to study kind of crap.

For MOST programs you need a bachelors as the programs are master's degrees, though there are a couple of Bachelor's programs left out there and like 2-3 certificate programs. However, these are going by the wayside and being phased out. Some states and many jobs now require a masters to even practice. What are your grades like? What prior medical experience do you have? Most PA schools are very big on having some real life experience. Competition is stiff to get in, and just getting stiffer.

If you want to get a good idea where you stand and learn how to better improve your application go to http://www.physicianassistantforum.com/forums/forum.php and hang out a bit. I would be glad to help you out however I can, so feel free to PM me.

FlyingHunter
12-12-12, 19:57
I agree its a great glass, I love learning how everything is connected. The only issue for me with all these classes seams to be every time I learn about some new disease I think i have it.:D.

That's a funny flashback...I attended Medical College of Georgia and felt the same way in my pathophysiology class!

FWIW...If you ever consider pursuing a Doctorate in Physical Therapy, pm me, I'll be happy to help you any way I can. I do ortho/sportmed for my daily bread...rehab for rotator cuff tears, knee ligament reconstructions etc...

I wish you the very best with your studies and the pursuit of your career.

acaixguard
12-25-12, 21:03
Thought I'd piggyback off this thread since this applies to me now too.

I graduated 13 years ago with a bachelor's in Comp Sci, and have been working in IT since then. I've always HATED it with a passion, even while in school, and finally made the recent decision to make a career change.
I am having a hard time deciding if I want to go for an RN, or go to paramedic school in a year or two. As others have mentioned, you get higher pay and more career options with the RN. However, I'm a bit of a "street" person, and being out and about, with the occasional adrenaline rush is more of my thing, which is why the job description of a medic seems a bit more fitting for my persona.
Currently, I am a new volunteer in my local EMS squad, and begin EMT training in 2 weeks. I also am registered with my county college to begin classes for my nursing pre-reqs. Thankfully, since I already have a degree, I don't have to take too many pre-reqs. Also, the paramedic programs here and share most of the same pre-req courses as nursing, so that's a plus.
My one hesitation with nursing is, I hear the competition for entry level jobs out of school is pretty stiff these days, especially if you don't have a BSN. Right now, I can only go for an associates in nursing. I can likely pursue a BSN later on down the road, but for a few reasons, it's out of the question right now. Not sure if entry level paramedic jobs are as competitive.
I have at least a year before I have to decide for sure what program to pursue, so my main priority now is to do well in my pre-req courses. However, any input between the two programs is welcome.
Thanks in advance!

aleaddict
12-25-12, 22:25
Either PA or nurse anesthetist would be a good choice, especially considering the current healthcare climate. I am a perfusionist (CCP) but would have been just as happy as a physician assistant. More options as a PA than CCP.

medicmiles
01-12-13, 21:15
Definitely PA you more of an opportunity to work in an ER setting at least in my area. It's mostly Dr s and PA s calling the shots.

drsal
01-12-13, 21:22
I concur with the other docs here, PA. All the best!

chuckman
01-18-13, 15:37
In all seriousness though, PA is a great route (though I am biased, as I am in PA school @ duke right now)..

I work at Duke...PM me.

chuckman
01-18-13, 15:44
I am having a hard time deciding if I want to go for an RN, or go to paramedic school in a year or two. As others have mentioned, you get higher pay and more career options with the RN. However, I'm a bit of a "street" person, and being out and about, with the occasional adrenaline rush is more of my thing, which is why the job description of a medic seems a bit more fitting for my persona.
Currently, I am a new volunteer in my local EMS squad, and begin EMT training in 2 weeks. I also am registered with my county college to begin classes for my nursing pre-reqs. Thankfully, since I already have a degree, I don't have to take too many pre-reqs. Also, the paramedic programs here and share most of the same pre-req courses as nursing, so that's a plus.
My one hesitation with nursing is, I hear the competition for entry level jobs out of school is pretty stiff these days, especially if you don't have a BSN. Right now, I can only go for an associates in nursing. I can likely pursue a BSN later on down the road, but for a few reasons, it's out of the question right now. Not sure if entry level paramedic jobs are as competitive.
I have at least a year before I have to decide for sure what program to pursue, so my main priority now is to do well in my pre-req courses. However, any input between the two programs is welcome.
Thanks in advance!

As you well know, medic, nurse...two different animals but related because they are still animals. I was a medic for 15 years, now a RN for 10. You know where the money is better, and nursing has way more job mobility and career options (clinical, nursing, education, research, case management, flight, management, etc.). I loved being in a bus, but made $3/hour more my first nursing gig than I did after 13 years as a senior paramedic.

Another education option is to get the ADN or diploma and do a RN-to-BSN program. Being a medic is way more fun and in a lot of ways much easier, but is physically and mentally more taxing and has less career growth. Nursing has more money, more career options and more mobility, less 'fun' but just as interesting.

ramairthree
01-29-13, 13:36
If you go the RN route,
I would rec. the ADN, pass NCLEX,
and work full time as an RN while finishing the BSN part time.

It worked out great for my wife and four others I know.

As for Organic,
the big cut for classes for a competetive MD school are how you do in that and also Calc.

some do not even require calc, if you are lucky enough to be a state resident of a state with serious resident preferences or looking at a school people dont like the location, etc. lower organic and mcat scores are not as big an issue.

WillBrink
01-29-13, 15:53
If you go the RN route,
I would rec. the ADN, pass NCLEX,
and work full time as an RN while finishing the BSN part time.

It worked out great for my wife and four others I know.

As for Organic,
the big cut for classes for a competetive MD school are how you do in that and also Calc.


Not biochem? You need two semesters of org to take biochem and the med schools I looked at all required nothing below a B in biochem. Biochem is often the class used to "separate the wheat from the chaff" as my proff said in the beginning of biochem to all the wanna be med students in the class.

I know some med schools have dropped biochem as a rec (which I consider a damn shame and a mistake) but i believe the more competitive schools still require biochem.

Org was a PITA to be sure, but it was no biochem in the neuron stressing "how the fu&% am I supposed to possibly know and memorize that much information??!!" dept I thought. :cool:

After biochem I took a course called Biochemical and Molecular Physiology with Dr Dressler. I'll never forget the name of the course or the proffs name as it was almost the death of me, but I have to admit, fascinating stuff.

Ledanek
01-31-13, 10:01
get a PA program as someone has stated, your insurance is covered by the dr you work for, because 2014, insurance will be overhauled, and a large community dr are NOT happy with it. So get in the know about insurance.

Work your way up to Medicine, why stop now, your young. Suggest Radiologist...Imaging Medicine supports a Hospital like a huge rock. They get patients in and out as fast as ER can bring them in. Reimbursements are more, though slower in return, but that's in the administration's court.

ICANHITHIMMAN
01-31-13, 10:43
get a PA program as someone has stated, your insurance is covered by the dr you work for, because 2014, insurance will be overhauled, and a large community dr are NOT happy with it. So get in the know about insurance.

Work your way up to Medicine, why stop now, your young. Suggest Radiologist...Imaging Medicine supports a Hospital like a huge rock. They get patients in and out as fast as ER can bring them in. Reimbursements are more, though slower in return, but that's in the administration's court.

I suppose in a round about way I am working up now. Radiologist is a no go for where I live at present the market is very saturated! The only advantage I can see having in nursing in my area is the people I know. If you guys have any links to scholarships for vets let me know, my lab partner in micro is looking at PA school and any moneys will help him out.

Shoulderthinggoesup
02-01-13, 22:31
Scholarships for PA school pretty much don't exist. There is are random 500$ or 1k ones for weird stuff.... but that is it. The only real scholarship is for people that want to go into undeserved primary care: http://nhsc.hrsa.gov/loanrepayment/

ramairthree
02-06-13, 15:59
The classic premed requirements are:
-Chemistry with lab (one year)

-Organic chemistry with lab (one year)

-Biology with lab time (one year)

-Physics with lab time (one year)

-Calculus (one year)

(and many say a year of English)

The classic cut where a lot of premeds find they are no longer premed is Organic.

Calculus is similar.

The trend seems to be if you are average or bright and work hard you can memorize plenty of stuff to do well in the other courses. But you have to be at least bright, just working hard won't make up for stuff that takes X amount of mental horsepower to understand or get vs. just memorize.

There are plenty of exceptions, special programs, etc. but in general schools that don't require the calc, don't look hard at the organic, accept courses without labs, etc. are the less competitive schools.

Biochem is recommended at many and required by a few, but it is a crap shoot. Some places that require it are top notch, others are not, and not all top places require it. It is generally considered a memorize a bunch of stuff vs. hard to understand type of course. While you had one professor say it is the make or break course, you will find the vast majority give that distinction to Organic.

Different people will find different courses harder in different ways based on likes, teachers, etc. P-Chem may be easier than Orgo at one school, but at another it will be taught by some crusty Swiss German that hates lazy, stupid American undergrads and can make the smartest chem majors cry. I am taking it you had a Biochem guy that liked putting the squeeze on undergrads.

The concept of Med School being very competitive in the US is different than in some other countries, and the pre reqs were not supposed to measure if someone could pass med school, but "weed in" personnel that would have a very high quality density.

GPA is also taken with a grain of salt. A competitive school is not going to consider the 4.0 science GPA from Bill's College and Bait shop the same as a 3.8 in Biological Engineering from MIT.

At the end of the day, like many things, the standards fluctuate. A guy whose record will not get him an interview at a top notch school or even his good state school may be a shoo in at another school. There are plenty of options so now an average smart, hard working guy, with no special admission considerations can find a place to become a physician in the US even if he got a 21 on his MCATS. But, a two standard deviations above the mean 35 MCAT guy may not get in at the school he really wants to go to or the location he wants to go to if it is super competitive.

Many do not consider the times right when deciding. They say PA is four years, CRNA is 6, MD is 8, etc. but do not take into account programs changing length, experience requirements, internship, residency, etc. Make sure you weigh in everything accurately. Job satisfaction and salary can also be very different. CRNAs and NP may have a similar length of training, but academic intensity, pay, and job satisfaction tend to be dissimilar. A family practice doc and an emergency medicine doc may have the same total years of training and education, but very different pay and schedules. Make sure you get a feel for that sort of thing also. I know someone that spent her entire life wanting to be a Pediatrician, and after 8 years of college and med school, was halfway through residency realizing she hated being a pediatrician.

Also, if you go the physician route, the "rank" of the school you went to can be irrelevant. A guy that went to med school at Ponce, Howard, or DO school then did an Ortho residency at a county hospital is going to make far more money than a Pediatrician, PM doc, or FP that went to med school at Columbia and did a residency at Mayo or Mass General. If a guy wants to be a program director at a prestigious institution or head a department at a big house, then where they got there MD and where they did their residency may be important. Where you want to do residency can come into play also. You are probably not going to get into an Orthopedic Surgery residency at Mayo Rochester if you went to med school at Oklahoma State, but then again if you played in the NFL for a while after your undergrad maybe you will.

Also, there are other considerations. You may have the juice to get into a top quartile school, but the location of another school is where you decide to go even though it is only a top third school, etc. The best applicant in the world may choose a more rural med school over an urban one, or get offered incentives for a different school or something.

Think hard about all the choices and good luck.

Hmac
02-06-13, 17:26
Pretty good overview.

JC1990
03-10-13, 23:07
Good read.

I am 22, was going for English Education, and with 3 years in decided I hated it.

I take my CNA test this week, and am going to work as a CNA while getting my BSN, and eventually hope to become a NP. The first year transferred, so I have 3 years left to get my RN. I think I am going to be much happier in the medical field.

Ligament
03-11-13, 01:56
I'd suggest being a PA. You get to do cool stuff within your scope of training, but don't take on any medicolegal risk.

Nurse Practitioners are absolutely incompetent at anything other than the most basic of primary care duties. They don't know what they dont't know, and don't want to know that. Trying to play doctor with zero of the training.

Hmac
03-11-13, 07:09
I'd suggest being a PA. You get to do cool stuff within your scope of training, but don't take on any medicolegal risk.

Nurse Practitioners are absolutely incompetent at anything other than the most basic of primary care duties. They don't know what they dont't know, and don't want to know that. Trying to play doctor with zero of the training.

My experience over many years has been that that's true to some degree of many or most "physician extenders". But I do agree, I'm more comfortable with PAs in my particular line of work (surgery).

There's going to be a lot of state, local and institutional variation in the duties, skills and capabilities of PAs and NPs. I would be more concerned about the individual's training and experience than their certification or licensure.

montanadave
03-11-13, 09:32
I'd suggest being a PA. You get to do cool stuff within your scope of training, but don't take on any medicolegal risk.

Nurse Practitioners are absolutely incompetent at anything other than the most basic of primary care duties. They don't know what they dont't know, and don't want to know that. Trying to play doctor with zero of the training.

A rather sweeping condemnation with no data to back it up.

It's the individual, not the alphabet soup after the name. I've met good and bad of both persuasions.

Sensei
03-11-13, 23:57
Nurse Practitioners are absolutely incompetent at anything other than the most basic of primary care duties. They don't know what they dont't know, and don't want to know that. Trying to play doctor with zero of the training.

Could you please let us know your background and qualifications to make such a statement. My practice of emergency medicine employes NP's and I supervise them when I practice clinically (granted this is only 10-20% of my time these days). I also interact with NP's from our trauma, hospitalist, cardiology, neurosurgery, and intensivist services when they come down to admit our patients. Granted, being a NP is not the route that I took. However, I just drank a pot of coffee and am on a mission tonight to dispel all the BS on the internet. For some reason, your post is at the top of my list. :dirol:

Ligament
03-12-13, 01:33
duplicate post sorry...

Ligament
03-12-13, 01:34
Could you please let us know your background and qualifications to make such a statement. My practice of emergency medicine employes NP's and I supervise them when I practice clinically (granted this is only 10-20% of my time these days). I also interact with NP's from our trauma, hospitalist, cardiology, neurosurgery, and intensivist services when they come down to admit our patients. Granted, being a NP is not the route that I took. However, I just drank a pot of coffee and am on a mission tonight to dispel all the BS on the internet. For some reason, your post is at the top of my list. :dirol:

I'm a medical physician double board certified by the American Board of Medical Specialties and ACGME residency and ACGME fellowship trained in a very competitive field. I am in full time clinical practice active staff at three major hospitals in a very metropolitan area.

Unlike you, I personally perform all consults, procedures, and followups on my patients, as my job cannot be performed by "mid-levels."

You do realize, I hope, that you are contributing to the demise of our profession by supervising NPs that are already practicing independently and will someday replace physicians as they are cheaper and do not question the treatment guidelines laid down by the insurance companies and government? Obama and his Marxist friends would like nothing better than to replace physicians who are trained well enough to question their dictums and hence roadblock their agendas.

Don't believe this can happen? Look at what the Anesthesiologists did to themselves by training and supervising the CRNAs.

I actually have two NPs that are close personal friends and the questions they ask me are so basic it is scary. However, they are my friends first and foremost so I'm not harsh on them.

And to reply to montanadave; I'm sure you mean well but no, an individual's intentions or personality mean nothing when they have no appropriate medical training to do the task. Get the training first, then those factors you mention come into play. This would apply foremost to the NPs who consider themselves competent in the independent practice of medicine rather than the PAs who understand their duties and are proud to perform them.

Hmac
03-12-13, 05:06
I'm a medical physician double board certified by the American Board of Medical Specialties and ACGME residency and ACGME fellowship trained in a very competitive field. I am in full time clinical practice active staff at three major hospitals in a very metropolitan area.

Unlike you, I personally perform all consults, procedures, and followups on my patients, as my job cannot be performed by "mid-levels."

You do realize, I hope, that you are contributing to the demise of our profession by supervising NPs that are already practicing independently and will someday replace physicians as they are cheaper and do not question the treatment guidelines laid down by the insurance companies and government? Obama and his Marxist friends would like nothing better than to replace physicians who are trained well enough to question their dictums and hence roadblock their agendas.

Don't believe this can happen? Look at what the Anesthesiologists did to themselves by training and supervising the CRNAs.

I actually have two NPs that are close personal friends and the questions they ask me are so basic it is scary. However, they are my friends first and foremost so I'm not harsh on them.

And to reply to montanadave; I'm sure you mean well but no, an individual's intentions or personality mean nothing when they have no appropriate medical training to do the task. Get the training first, then those factors you mention come into play. This would apply foremost to the NPs who consider themselves competent in the independent practice of medicine rather than the PAs who understand their duties and are proud to perform them.

Oops. Bullshit meter just pegged.

Sound more like a PA with NP envy.


/

ICANHITHIMMAN
03-12-13, 06:12
I'm a medical physician double board certified by the American Board of Medical Specialties and ACGME residency and ACGME fellowship trained in a very competitive field. I am in full time clinical practice active staff at three major hospitals in a very metropolitan area.

Unlike you, I personally perform all consults, procedures, and followups on my patients, as my job cannot be performed by "mid-levels."

You do realize, I hope, that you are contributing to the demise of our profession by supervising NPs that are already practicing independently and will someday replace physicians as they are cheaper and do not question the treatment guidelines laid down by the insurance companies and government? Obama and his Marxist friends would like nothing better than to replace physicians who are trained well enough to question their dictums and hence roadblock their agendas.

Don't believe this can happen? Look at what the Anesthesiologists did to themselves by training and supervising the CRNAs.

I actually have two NPs that are close personal friends and the questions they ask me are so basic it is scary. However, they are my friends first and foremost so I'm not harsh on them.

And to reply to montanadave; I'm sure you mean well but no, an individual's intentions or personality mean nothing when they have no appropriate medical training to do the task. Get the training first, then those factors you mention come into play. This would apply foremost to the NPs who consider themselves competent in the independent practice of medicine rather than the PAs who understand their duties and are proud to perform them.

Thanks for posting your opinion, I is not very in keeping with the theme of my thread. However when you ask a question on an open forum you are bound to get your fair share of undesirable responses. I know all the medical professional here who have taken the time to respond to my thread and help me in my decision will not be offended by how stupid you think they all are, because they have not achieved your status and accreditation. I only hope your "friends" are more forgiving when they learn of the lowly status they occupy beneath you. I will have to ask at this point, that you do not offer your free advice again and I ask this as your "friend".

R/
Jon Morehouse

Ligament
03-12-13, 14:18
Thanks for posting your opinion, I is not very in keeping with the theme of my thread. However when you ask a question on an open forum you are bound to get your fair share of undesirable responses. I know all the medical professional here who have taken the time to respond to my thread and help me in my decision will not be offended by how stupid you think they all are, because they have not achieved your status and accreditation. I only hope your "friends" are more forgiving when they learn of the lowly status they occupy beneath you. I will have to ask at this point, that you do not offer your free advice again and I ask this as your "friend".

R/
Jon Morehouse

You are correct, off topic, and you have my apologies. I was responding to Sensei who asked me very specific questions about my credentials, to which I responded. I see neither Sensei's question nor my reply helped you in any way. These should have been PMs between us. :)

You'll see in my first post to this thread I attempted to offer you useful advice. V/r

Hmac
03-12-13, 14:52
I'm a medical physician double board certified by the American Board of Medical Specialties and ACGME residency and ACGME fellowship trained in a very competitive field.

What does this even mean? I know a lot of doctors. I've never heard one describe themselves like this. "Medical physician"? "Double board certified by ABMS and ACGME"?

I doubt that you're a doctor, in other words.

I risk going further off topic in order to make sure that no one takes your opinion seriously.

Sensei
03-13-13, 00:17
What does this even mean? I know a lot of doctors. I've never heard one describe themselves like this. "Medical physician"? "Double board certified by ABMS and ACGME"?

I doubt that you're a doctor, in other words.

I risk going further off topic in order to make sure that no one takes your opinion seriously.

I took from his posting that he was double boarded, and completed an ACGME approved residency and fellowship. However, his wording was curious. Most of us simply state our board certifications when addressing a colleague since completion of residency is a prerequisite. For example, I am board certified by ABEM and a Fellow of the American College of Emergency Physician (FACEP). You see, not that hard - no cloak and dagger. I've never known a physician to specify that their residency is ACGME accredited since all US residency programs are to the best of my knowledge. I've also never known someone to solidify their credentials by telling me that they practice in a very competitive field.

Having said all that, his statements about the effects of NPs (or any mid-level provider) on our job market were way off base. There is not going to be any physicians starving due to lack of employment anytime in the near future. There is more than enough work to go around and everybody can make some money if they are willing to work hard. In fact, PAs and NPs are vital to the profitability of our practice and work-life balance.

My personal preference had I not gone to medical school would be PA school. The biggest problem that I have with NP and nurse anesthesia programs is the requirement to be a practicing nurse for a period of time. On the other hand, PA programs have a more direct career path.

I do have one warning for those considering a career in medicine. There is a new player in town and you better know how to deal with him. That player is patient satisfaction. Virtually every MD, PA, NP, etc. gets patient satisfaction scores and many times those scores influence your pay check. It is not only your insurance paying patients that get them, so will the uninsured, Medicaid, and drug seekers. Most of my partners will secretly admit that this dynamic dominates every case that they see. They fret over how to manage patient expectations for tests, antibiotics, drugs, etc. They get pissed when someone returns a survey with all 1's (5 being the highest) because the did not give the patient Percocet.

I handle this issue with a honey badger attitude -I don't give a shit. I'm polite to everybody, but I'm not a candyman when it comes to drugs unless you have a real, objective reason to need them. I also don't give little Johnny a Z-pack for his sniffles because mommy expects it. I do my best to explain the situation, and act on what I believe is in the best interests of my patients and the broader community. If I get a bad survey that costs me some money, oh well. Many people cannot come to terms with this issue.

Hmac
03-13-13, 06:37
I took from his posting that he was double boarded, and completed an ACGME approved residency and fellowship. However, his wording was curious. Most of us simply state our board certifications when addressing a colleague since completion of residency is a prerequisite. For example, I am board certified by ABEM and a Fellow of the American College of Emergency Physician (FACEP). You see, not that hard - no cloak and dagger. I've never known a physician to specify that their residency is ACGME accredited since all US residency programs are to the best of my knowledge. I've also never known someone to solidify their credentials by telling me that they practice in a very competitive field.




The way I read it, he thinks that being double boarded means that he's board certified by both the ABMS and the ACGME. Neither of those are bodies that accredit physicians.

His whole presentation doesn't ring true to me, including his attitudes about mid-level practitioners. The creeping "doctorization" of advanced practice nurses is a market response to the increasingly severe physician shortage. It doesn't threaten the medical profession.

ICANHITHIMMAN
03-13-13, 06:43
I took from his posting that he was double boarded, and completed an ACGME approved residency and fellowship. However, his wording was curious. Most of us simply state our board certifications when addressing a colleague since completion of residency is a prerequisite. For example, I am board certified by ABEM and a Fellow of the American College of Emergency Physician (FACEP). You see, not that hard - no cloak and dagger. I've never known a physician to specify that their residency is ACGME accredited since all US residency programs are to the best of my knowledge. I've also never known someone to solidify their credentials by telling me that they practice in a very competitive field.

Having said all that, his statements about the effects of NPs (or any mid-level provider) on our job market were way off base. There is not going to be any physicians starving due to lack of employment anytime in the near future. There is more than enough work to go around and everybody can make some money if they are willing to work hard. In fact, PAs and NPs are vital to the profitability of our practice and work-life balance.

My personal preference had I not gone to medical school would be PA school. The biggest problem that I have with NP and nurse anesthesia programs is the requirement to be a practicing nurse for a period of time. On the other hand, PA programs have a more direct career path.

I do have one warning for those considering a career in medicine. There is a new player in town and you better know how to deal with him. That player is patient satisfaction. Virtually every MD, PA, NP, etc. gets patient satisfaction scores and many times those scores influence your pay check. It is not only your insurance paying patients that get them, so will the uninsured, Medicaid, and drug seekers. Most of my partners will secretly admit that this dynamic dominates every case that they see. They fret over how to manage patient expectations for tests, antibiotics, drugs, etc. They get pissed when someone returns a survey with all 1's (5 being the highest) because the did not give the patient Percocet.

I handle this issue with a honey badger attitude -I don't give a shit. I'm polite to everybody, but I'm not a candyman when it comes to drugs unless you have a real, objective reason to need them. I also don't give little Johnny a Z-pack for his sniffles because mommy expects it. I do my best to explain the situation, and act on what I believe is in the best interests of my patients and the broader community. If I get a bad survey that costs me some money, oh well. Many people cannot come to terms with this issue.

WOW thanks for posting that, I had no idea. Its funny to think that there are people out there who act like this. Its just so foreign to my mind set that I cant fathom it. I cant see this being an issue with other medical providers but with admin types. Can you explain how the survey came to be?

montanadave
03-13-13, 07:52
I handle this issue with a honey badger attitude -I don't give a shit. I'm polite to everybody, but I'm not a candyman when it comes to drugs unless you have a real, objective reason to need them. I also don't give little Johnny a Z-pack for his sniffles because mommy expects it. I do my best to explain the situation, and act on what I believe is in the best interests of my patients and the broader community. If I get a bad survey that costs me some money, oh well. Many people cannot come to terms with this issue.

Excellent post. And I particularly admire the sentiment expressed in the section quoted above.

The incestuously dependent yet paradoxically mutually assured destructive dynamic which currently binds the triad composed of the medical-insurance-legal communities has to be broken if we are ever going to arrest the unsustainable inflation of healthcare costs and provide effective affordable care. And putting the brakes on unnecessary treatments while concurrently educating the public (who ultimately bear the cost, one way or the other) is fundamental to making this transition.

ramairthree
03-13-13, 09:22
"I'm a medical physician double board certified by the American Board of Medical Specialties and ACGME residency and ACGME fellowship trained in a very competitive field."


It is worded a little odd, but I take it to mean that he is a U.S. educated MD, did an accredited MD residency and fellowship vs. AACOM Osteopathic programs, and has real ABMS board certification vs. the ABPS boards.

When a physician talks about the "doctorization" of other medical fields, they usually are referring to degree creep.

A physical therapist in 1990 graduated with a B.S. degree. Then it was an M.S. without any significant addition of clinical training. Now it is a Doctor of Physical Therapy. There have always been doctors that were not physicians, be it Optometry, Podiatry, Chiropractic, etc. But now there are doctorate degrees for PAs, Nurses, Pharmacists, etc.

Sometimes when a physician talks about doctorization, they may mean those fields wanting full autonomy and a complete lack of physician oversight. There is significant economic motivation to get as many people seen by personnel with less years of academic and clinical training as possible.

In what is hopefully M4Carbine type speak, Not everyone is a HRT guy vs. local LE, or Operator vs. ODA or Ranger Bn guy, or boat team guy vs. SEAL, but each has a very valid and valuable role.

The guy is trying to make a life affecting decision. Accurate, objective information in a non-offensive manner will serve him best.

Royalflush
03-13-13, 11:01
2 words: Veterinary medicine :-)
Much less political oversight, no obama-puppy-care. Nearly identical education compared to MD. Animals don't make up symptoms. lots of versatility with your doctorate. who doesn't want to play with puppies all day.
as far as money goes: you can't make a lot IF you work as an associate for someone- you'll probably max out at 100K after a few years of experience as a country average. pretty easy to own your own practice which will put you at $150K and up. Industry vets also make a pretty decent living. Specialists start at $150K and up.
AVMA provides malpractice insurance for CHEAP compared to what an MD would pay.
No real breech of ethics if you want to date patients owners :-)

Sensei
03-13-13, 12:51
WOW thanks for posting that, I had no idea. Its funny to think that there are people out there who act like this. Its just so foreign to my mind set that I cant fathom it. I cant see this being an issue with other medical providers but with admin types. Can you explain how the survey came to be?

This article in Forbes is one of the best summaries of where we are at in terms of patient satisfaction.

http://www.forbes.com/sites/kaifalkenberg/2013/01/02/why-rating-your-doctor-is-bad-for-your-health/

The fact that it came from outside of our industry means that the problem may finally start getting mainstream attention.

In the spirit of full disclosure, I'm leaving routine clinical practice to pursue an opportunity with a federal LE agency. At age 37, I've been working at very large emergency departments for 10 years and reached the top of my clinical game - time to try something new and expand my skillset. I'll still work part time a couple of shifts per month (thank you furlough!) to maintain my credentials.

The point of this is that medical school gives you lots of opportunities to do fun shit IF you think outside of the box AND you are not wedded to a particular standard of living/salary. For example, I got to customize deployments to IQ and Afghanistan with hardcore units only because I brought the right medical skills to the table. I have friends who are PAs for the 75th, docs for SF groups, and the medical director for NASA. All use their medical credentials to live very full lives in non-traditional practice environments. In other words med school, PA school, etc are great ways to have a blast if you are willing to work.

streck
03-13-13, 13:06
This article in Forbes is one of the best summaries of where we are at in terms of patient satisfaction.

http://www.forbes.com/sites/kaifalkenberg/2013/01/02/why-rating-your-doctor-is-bad-for-your-health/

Very interesting and enlightening. Thank you for posting....

Ligament
03-13-13, 14:40
The way I read it, he thinks that being double boarded means that he's board certified by both the ABMS and the ACGME. Neither of those are bodies that accredit physicians.

His whole presentation doesn't ring true to me, including his attitudes about mid-level practitioners. The creeping "doctorization" of advanced practice nurses is a market response to the increasingly severe physician shortage. It doesn't threaten the medical profession.

The American Board of Medical Specialties (http://www.abms.org/) is comprised of many other specialty boards. The ABMS coordinates certification, test taking, etc for them.

The ABMS handles certification for twenty something specialties (not all specialties), two of which are mine.

The American College of Graduate Medical Education (http://www.acgme.org/acgmeweb/) accredits internships, residencies, and fellowships. Completing an ACGME accredited fellowship allows one to be certified by the ABMS in a medical subspecialty. There are "private" fellowships that are not ACGME accredited, and therefore one cannot become ABMS board certified from such fellowships. Hence, I specified I completed an ACGME accredited fellowship.

One can also be "elected" as a fellow of various specialty organizations, such as becoming a Fellow of the American College of Surgeons (FACS). This is not the same as completing a fellowship (which is the highest level of training for physicians), but rather this is an honorary title.

There is no way to prove to you fellas that I am indeed a physician as long as I choose to maintain some anonymity on this forum. The medical professional community in my area is absurdly anti-self protection and anti-second amendment and prejudiced against anybody choosing to exercise their second amendment rights. I'm in the surgery center right now about to start a case, and the nurses are calling me "Doctor," so I'm pretty certain I'm a physician.

Hmac
03-13-13, 14:46
Sometimes when a physician talks about doctorization, they may mean those fields wanting full autonomy and a complete lack of physician oversight. There is significant economic motivation to get as many people seen by personnel with less years of academic and clinical training as possible.



We're seeing that in Minnesota now. There's a bill before the legislature that would do away with collaborative agreements and allow full prescriptive authority to NP and CRNAs. Frankly, I don't have a problem with that. At the facilities where I practice, it will mean that they will move from the Allied Health Professions staff to the Medical Staff and will be held to the same oversight and QA process as physicians - quality of care will be maintained. That assumes, of course, that the hospital or any of the practice groups would hire an "Advanced Practice" nurse under those circumstances. That's unlikely at least for the time being. Most likely, if the bill passes, the terms of employment will still require collaboration and oversight and will continue to do so as long as we don't become desperate for mid-levels. Which could happen someday sooner or later. As to CRNAs, Minnesota is an "opt out" state. Physician supervision isn't required by law, but since the CRNAs work for the hospital, they are most definitely subject to oversight by the Chief of Anesthesia.



This article in Forbes is one of the best summaries of where we are at in terms of patient satisfaction.

http://www.forbes.com/sites/kaifalkenberg/2013/01/02/why-rating-your-doctor-is-bad-for-your-health/

The fact that it came from outside of our industry means that the problem may finally start getting mainstream attention.



We've been using Press Ganey for about 6 years for the facility as a whole and for individual physicians. Frankly, in my administrative role, I find it to be pretty interesting and useful for the facility. We do occasionally see opportunities for improvement in patient management and I've used it on a number of occasions to bolster my arguments for facility and training upgrades. I can honestly say that I have yet to see any physician overtly altering their practice in a way that is detrimental to patient care based on their Press Ganey scores.

Anyway, Press Ganey is only one of a whole host of such quality measures that I deal with regularly. We have SCIP, NSQIP, MBSAQIP, and a few other benchmarking QIP's and at least a dozen people whose sole job is data entry and management. And we have an ongoing in-house "report card" system that tracks a number of performance metrics for every physician on staff. I don't know how much it improves patient care, but it sure does cost our patients a lot of money.

These days, physician performance is tracked every which way. IMHO, that's a good thing in large part, at least in facilities where it's utilized constructively. I am skeptical, however, that the Federal government will be capable of doing so as the process continues to mature.

Hmac
03-13-13, 14:49
I'm in the surgery center right now about to start a case, and the nurses are calling me "Doctor," so I'm pretty certain I'm a physician.

Well, that's good enough for me. Unless you're one of the new breed of CRNAs, PAs, NPs, or Pharmacists that now have a "doctoral" degree (j/k).

Sensei
03-13-13, 16:31
We're seeing that in Minnesota now. There's a bill before the legislature that would do away with collaborative agreements and allow full prescriptive authority to NP and CRNAs. Frankly, I don't have a problem with that. At the facilities where I practice, it will mean that they will move from the Allied Health Professions staff to the Medical Staff and will be held to the same oversight and QA process as physicians - quality of care will be maintained. That assumes, of course, that the hospital or any of the practice groups would hire an "Advanced Practice" nurse under those circumstances. That's unlikely at least for the time being. Most likely, if the bill passes, the terms of employment will still require collaboration and oversight and will continue to do so as long as we don't become desperate for mid-levels. Which could happen someday sooner or later. As to CRNAs, Minnesota is an "opt out" state. Physician supervision isn't required by law, but since the CRNAs work for the hospital, they are most definitely subject to oversight by the Chief of Anesthesia.




We've been using Press Ganey for about 6 years for the facility as a whole and for individual physicians. Frankly, in my administrative role, I find it to be pretty interesting and useful for the facility. We do occasionally see opportunities for improvement in patient management and I've used it on a number of occasions to bolster my arguments for facility and training upgrades. I can honestly say that I have yet to see any physician overtly altering their practice in a way that is detrimental to patient care based on their Press Ganey scores.

Anyway, Press Ganey is only one of a whole host of such quality measures that I deal with regularly. We have SCIP, NSQIP, MBSAQIP, and a few other benchmarking QIP's and at least a dozen people whose sole job is data entry and management. And we have an ongoing in-house "report card" system that tracks a number of performance metrics for every physician on staff. I don't know how much it improves patient care, but it sure does cost our patients a lot of money.

These days, physician performance is tracked every which way. IMHO, that's a good thing in large part, at least in facilities where it's utilized constructively. I am skeptical, however, that the Federal government will be capable of doing so as the process continues to mature.

I have no doubt that patient satisfaction surveys provide some useful data. The question remains whether the unintended consequences of coupling physician compensation to this metric outweigh the benefits. That Forbes article makes a strong case that the downside is significant.

I can tell you that tying compensation and satisfaction has been an unmitigated disaster when it comes to emergency department resource use (antibiotics, controlled substances, testing, and admissions). Think about it - almost 40% of ED care is EMTALA driven care to the uninsured (Medicaid + Self-pay). These people also get surveys even though they have absolutely zero financial stake in their care or incentive to conserve resources. None of them will ever come close to paying a co-pay, much less the entire bill. Yet, we incentivize physicians to meet their expectations which are often unreasonable. Personally, I consider lack of payment on a bill to be the equivalent of theft. Giving these people a satisfaction survey is akin to asking a shoplifter at Walmart how they enjoyed their stealing experience after they made off with a TV.

g-rap
03-13-13, 20:06
Late to the game, but ever think of Pharmacy school. Clinical Pharmacy in every specialization. Not revered as the other medical fields, but money is good, ins. is cheap, and not set in specialization (a acute care pharmacist can go work in retail no problem).

kiwi57
03-13-13, 20:10
Certainly interesting to watch the direction this thread has taken since I last posted. ICANHITHIMMAN, I hope your studies are going well and you’ve had a chance to shadow professionals in their everyday practice to get a glimpse of what might lay ahead.

There really is no wrong answer if you have a desire to care for people and help them to live better and longer through your efforts.

I wanted to clear up one point about PAs. To my knowledge, there is one accredited PA program that awards a clinical doctorate—a joint effort between the US Army and Baylor University. That’s out of 170+ PA programs. There is very little, if any, interest to follow the path of our nursing, pharmacy and other colleagues down the path of degree creep.

A summit in 2009 addressed the issue of the “clinical doctorate” and the conclusion was that a master’s degree should be the terminal degree for physician assistants (see JAMA Commentary http://jama.ama-assn.org/cgi/content/short/305/24/2571). There was no evidence that patient outcomes would be improved by adding more training to the already demanding education that PAs receive. We have a saying in the PA profession: “We already have a doctorate in medicine—it’s called an MD.” The implication is that if you want to be a doctor, get accepted to medical school. Docs are, and should remain, the captain of the ship.

I have practiced with and trained a lot of PAs in 30 years, and not one has wanted to practice independently. Even though I’ve run a clinic 120 miles from the nearest hospital (and my supervising docs), I’ve always respected and understood my dependent relationship with physicians. Are there PAs who make mistakes and don’t ask for help when they should, or are just bad apples? Of course—just like docs—and every other medical and nursing professional.

I consider myself incredibly lucky and privileged to have practiced medicine in partnership with my docs (one for over 20 years).

I apologize for the length of this post. Continued best wishes for your career ICANHITHIMMAN.

Kiwi57

Texas42
03-14-13, 03:23
2 words: Veterinary medicine :-)
Much less political oversight, no obama-puppy-care. Nearly identical education compared to MD. Animals don't make up symptoms. lots of versatility with your doctorate. who doesn't want to play with puppies all day.
as far as money goes: you can't make a lot IF you work as an associate for someone- you'll probably max out at 100K after a few years of experience as a country average. pretty easy to own your own practice which will put you at $150K and up. Industry vets also make a pretty decent living. Specialists start at $150K and up.
AVMA provides malpractice insurance for CHEAP compared to what an MD would pay.
No real breech of ethics if you want to date patients owners :-)

I nevered envied Vets to have to convince their clients that need to actually pay for their care or that that broken leg needs an xray. :)

chuckman
03-14-13, 17:57
Well, that's good enough for me. Unless you're one of the new breed of CRNAs, PAs, NPs, or Pharmacists that now have a "doctoral" degree (j/k).

Don't even get me freakin' started....

I am a RN, have a BSN, thought passively about NP school but because of the DNP I refuse to do it. It's because historically nursing has had an inferiority complex and want to see a horizontal relationship with physicians, DOs, whatever. They need 'doctor' to make themselves feel good, just as they need every specialty certification and credential behind their name on their badge.

I have two bachelor degrees, half of a masters degree from Duke, and teach every alphabet soup/merit badge course there is, and hold NREMTP and CCP. My badge has my name followed by RN.

Where I work is very aggressive regarding the DNP (one of the first universities offering it) and will pipeline clinically unsound nurses through the program just to get them on board. Disgusting.

Royalflush
03-14-13, 21:02
I nevered envied Vets to have to convince their clients that need to actually pay for their care or that that broken leg needs an xray. :)

Not as hard as you'd think. Average small animal appointment is $107. 3-4 appts per hour, and that general practice. Plus you'll be fully licensed to do surgery and medicine.
I applied to medical and vet school. Got in to both. Decided on vet school for lifestyle preferences.

And there are no "fake" veterinarians :-)

My wife is applying for a doctor of physical therapy program. I think it's quite a bit different than comparing an md to a dnp.
Especially since field of expertise have much less overlap. Thoughts? They might "deserve" to be called doctor since its harder for patients to confuse a physical therapist with an md. That's really my only issue with everyone in the hospital being called doctor.

Texas42
03-15-13, 02:51
Not as hard as you'd think. Average small animal appointment is $107. 3-4 appts per hour, and that general practice. Plus you'll be fully licensed to do surgery and medicine.
I applied to medical and vet school. Got in to both. Decided on vet school for lifestyle preferences.

And there are no "fake" veterinarians :-)

My wife is applying for a doctor of physical therapy program. I think it's quite a bit different than comparing an md to a dnp.
Especially since field of expertise have much less overlap. Thoughts? They might "deserve" to be called doctor since its harder for patients to confuse a physical therapist with an md. That's really my only issue with everyone in the hospital being called doctor.

Forgive the ignorant, poor Intern on overnight, call, but what does a Doctor of Physical Therapy do? Do they work "normal" PT/OT jobs?

PS: thanks for the info about Vet's. It is MUCH more difficult to get into vet school than Med school. There are what? 26 programs in the us?

Royalflush
03-15-13, 10:30
I can't claim to be an expert on the doctor of PT thing. My wife has struggled to make up her mind on what she wants to do when she grows up and kinda settled on the dot program. That being said- her reasons have nothing to do with the "doctor" title. She wishes it was still a 2 year masters program.
But what what I ASSUME a DPT does exactly what a PT with a masters degree OR a PT from 20 years ago with a bachelors degree does. Someone correct me if that is false?

And yeah the veterinary thing is cool. Because of ethical concerns related to use of human patients and cadavers- you get to learn a lot more, and much faster in the education process. But it is challenging! Most species share a lot of similar physiological concepts, but there are a ton of differences. Thus different animals experience different diseases, and are treated in different ways. So think about going to med school for 6 different species :-/

I would encourage anyone who is in undergrad, and interested in medicine of any sort to go by a local vet hospital. Veterinarians are typically MUCH more willing to let you get involved in learning/shadowing/ and even hands on experience while you are still in undergrad. Even if you decide to stick with humans- you can carry over some technical skills you may pick up at the veteranary clinic to your medical education.

It is a little harder to get in, but keep your grades up and kiss some ass. Very political process

Hmac
03-15-13, 11:06
Veterinarians are very cool. We have 3 dogs, 2 horses and deal with them a lot. I love talking with them about professional matters. I do some flexible endoscopy for a couple of them and have operated with one of them on a couple of my dogs over the years (did a spenectomy on one of my Golden Retrievers for hemangiosarcoma a couple of years ago). I am always amazed at the depth of their knowledge in such widely varying physiologies.

Having said that, my observation has been that veterinary medicine appears to be a calling for someone with a keen love of animals first and a love of medicine second.


I sent this to our large animal vet. He got a kick out of it.

http://ic2.pbase.com/o4/60/230460/1/149217254.ZE5isQpu.larsonhorses.jpg

Royalflush
03-15-13, 12:58
Shouldn't human medicine also be proceeded by a love of humanity followed by a love of medicine :-)
I'm just messin with ya. It is a much more relaxed setting to practice, I can say that for sure!
More than my love for animals is my appreciation for the relationship between owner and pet. But yeah, I do get pretty damn happy when I get to play with puppies!!!!

That picture is funny, I've seen similar t shirts. Sometimes it's really the way I feel about large animal medicine!

Royalflush
03-15-13, 12:58
Preceded*

Hmac
03-15-13, 14:56
Shouldn't human medicine also be proceeded by a love of humanity followed by a love of medicine :-)
I'm just messin with ya. It is a much more relaxed setting to practice, I can say that for sure!
More than my love for animals is my appreciation for the relationship between owner and pet. But yeah, I do get pretty damn happy when I get to play with puppies!!!!

That picture is funny, I've seen similar t shirts. Sometimes it's really the way I feel about large animal medicine!

My point was pretty much that usually, people don't say to themselves "I really love to cut living things up, I guess I'll be either a doctor or a veterinarian". More commonly, I think people embark on either of those grueling educational paths with an end goal in mind that fulfills one's expectations of what they want to accomplish with their education. Yes, my desire to be a doctor was preceeded by a desire help people. I mean, I love animals and all that, but my perception was I could make an impact that would be more meaningful to me by being a doctor. I do agree about the relaxed lifestyle, but I wasn't really looking for that either.

Royalflush
03-15-13, 15:21
I agree- everyone is looking for something different!!!!

I hope everyone is as lucky as us to figure out what it is they would be truly happy doing!
It was a really tough choice for me between the 2. I got in to 2 med schools and 1 vet school- tough choice for sure!

TylerD
06-04-13, 21:47
Hay guys I'm at a cross roads in my schooling. I'm 3 years out of the mil and have been going to school for a few years now. I want to go into the medical field but I cant decide which diciploine to focus and I need to start applying to programs.

I'm considering Nurse Practitioner or Physician Assistant I know some of you here are EMTs, Medics, Corpsman, PAs and Doctors can you tell me what you think? I want to do more trama related stuff so whats the best choice.

I dont want to go to medical school right now, I'm not even considering it at this point. I live in Rochester NY so there is no shortage of great schools in the area and with the GI bill its a no brainer.

Jon

Jon, it will most surely depend on how much you want to do. I just finished nursing school and I can tell you first hand what they teach is how to take care of stable patients and identify ones who aren't. I actually am enrolled in an EMT class now to see the unstable part of the patient census. I think its a good thing not only to be able to identify patients in distress, but also how to take care of them. If you aspire to be an NP, I would suggest trying to be a PA instead. With that being said, I want to work critical care. I feel in this area you really have to know your patients....... not just pass meds.

ICANHITHIMMAN
06-04-13, 22:20
Jon, it will most surely depend on how much you want to do. I just finished nursing school and I can tell you first hand what they teach is how to take care of stable patients and identify ones who aren't. I actually am enrolled in an EMT class now to see the unstable part of the patient census. I think its a good thing not only to be able to identify patients in distress, but also how to take care of them. If you aspire to be an NP, I would suggest trying to be a PA instead. With that being said, I want to work critical care. I feel in this area you really have to know your patients....... not just pass meds.

Thanks for the first hand info, I just got my 2 year done and have enough credits for a 4 year degree, lol SHIT. But that's due to my transfer credits and all the other courses I took that don't fit into my liberal arts health related degree. Had micro last term WOW that's hard! I don't know what I want to do now, the micro KICKED MY ASS and now I feel like an idiot, very defeating to say the least. I'm heading back to the Iron for the summer and I'm looking forward to a little cash in my pocket,just so afraid of getting stuck there.

Jon

TylerD
06-04-13, 22:24
Thanks for the first hand info, I just got my 2 year done and have enough credits for a 4 year degree, lol SHIT. But that's due to my transfer credits and all the other courses I took that don't fit into my liberal arts health related degree. Had micro last term WOW that's hard! I don't know what I want to do now, the micro KICKED MY ASS and now I feel like an idiot, very defeating to say the least. I'm heading back to the Iron for the summer and I'm looking forward to a little cash in my pocket,just so afraid of getting stuck there.

Jon

Don't let yourself feel down about classes. I have had my fair share of moments where I felt crazy, but you shake it off. Keep in mind these classes delve much farther into specifics you really don't need to know in all honesty. It may help when you have an patient with a staph infection and you are judging which generation cephalosporin they need..... but in my field you have a MD to make that call...... I feel the need to know so that if the MD makes a mistake I can be a patient advocate..... but that's what being an RN is all about.

Ouroborous
06-05-13, 01:29
Thanks for the first hand info, I just got my 2 year done and have enough credits for a 4 year degree, lol SHIT. But that's due to my transfer credits and all the other courses I took that don't fit into my liberal arts health related degree. Had micro last term WOW that's hard! I don't know what I want to do now, the micro KICKED MY ASS and now I feel like an idiot, very defeating to say the least. I'm heading back to the Iron for the summer and I'm looking forward to a little cash in my pocket,just so afraid of getting stuck there.

Jon

Hardest part of Micro was taking a culture from where the sun don't shine for me!

Seriously though, Micro is an intense class. I did it over a summer which may have been easier due to the total immersion but still finished with a lower but passing grade.

Much of that info really doesn't apply to RN's as much as it does to the Docs and Microbiologists in the lab IMO–I wouldn't take it too hard man. If you survived it, that's good enough.

williejc
06-08-13, 00:36
Some if not all national medical organizations view firearm ownership as a social evil, and this mindset filters down to some extent. My point is don't mention guns as one of your hobbies--on your application or during an interview. I promise you that academia is biased on this issue.

Schools in the South and Southwest may be cheaper. Cost of living is said to be lower here.

Hmac
06-08-13, 07:16
Some if not all national medical organizations view firearm ownership as a social evil, and this mindset filters down to some extent. My point is don't mention guns as one of your hobbies--on your application or during an interview. I promise you that academia is biased on this issue.


This is good advice for job/education interviews in medical fields as well as most other areas. IMHO, it's usually best not to bring up guns or shooting on school or employment applications. Note that it's also probably wise to make sure that these kinds of things don't show up on accessible forms of social media.

ICANHITHIMMAN
06-08-13, 08:21
Thanks guys, I have never had a job interview before, I was thinking about taking a class on it or something. It really makes me uneasy even though I may not shot it.

ICANHITHIMMAN
06-08-13, 08:27
Hardest part of Micro was taking a culture from where the sun don't shine for me!

Seriously though, Micro is an intense class. I did it over a summer which may have been easier due to the total immersion but still finished with a lower but passing grade.

Much of that info really doesn't apply to RN's as much as it does to the Docs and Microbiologists in the lab IMO–I wouldn't take it too hard man. If you survived it, that's good enough.

LOL you just need the right patient. I was in the ER at Portsmouth Naval Hospital back in 2001 as a patient. They were conducting tests etc bla, bla, blood from arms, wrist, fingers etc, I think it was about the third time I was asked to roll onto my side when I decided this is ****ing NUTS and my twisted sense of humor came out (I wont lie I was a little scared I was going to die). Anyhow the nurse, pa (a man)who ever proceeded to take the stool sample and place it on that little card. He said something to the effect of "sorry I know this is awkward" to which my reply was "hay as long as it was as good for you as it was for me were golden". The look on that guys face was worth the 10 hours stay in the ER! LOL still makes me chuckle

Ouroborous
06-08-13, 12:01
LOL you just need the right patient. I was in the ER at Portsmouth Naval Hospital back in 2001 as a patient. They were conducting tests etc bla, bla, blood from arms, wrist, fingers etc, I think it was about the third time I was asked to roll onto my side when I decided this is ****ing NUTS and my twisted sense of humor came out (I wont lie I was a little scared I was going to die). Anyhow the nurse, pa (a man)who ever proceeded to take the stool sample and place it on that little card. He said something to the effect of "sorry I know this is awkward" to which my reply was "hay as long as it was as good for you as it was for me were golden". The look on that guys face was worth the 10 hours stay in the ER! LOL still makes me chuckle

That is hilarious!

That dark sense of humor you have will serve you well in your career. Just don't say that to your patients if they're getting a culture taken by you!